CACI Study Questions

अब Quizwiz के साथ अपने होमवर्क और परीक्षाओं को एस करें!

Which client should the nurse report to the healthcare provider as needing a prescription for Rh Immune Globulin (RhoGAM)? A) Woman whose blood group is AB Rh-positive. B) Newborn with rising serum bilirubin level. C) Newborn whose Coombs test is negative. D) Primigravida mother who is Rh-negative.

Correct Answer: D RhoGAM is indicated during pregnancy for a woman who is Rh-negative or within 72 of birth of a Rh-positive infant (D). RhoGAM is not indicated for (A, B, and C).

What is the normal range for cardiac output?

The normal range for cardiac output to ensure cerebral blood flow and oxygen delivery is 4 to 8 L/min.

The nurse is assessing a client with bacterial meningitis. Which assessment finding indicates the client may have developed septic emboli? A) Cyanosis of the fingertips. B) Bradycardia and bradypnea. C) Presence of S3 and S4 heart sounds. D) 3+ pitting edema of the lower extremities.

A) Cyanosis of the fingertips. Septic emboli secondary to meningitis commonly lodge in the small arterioles of the extremities, causing a decrease in circulation to the hands (A) which may lead to gangrene. (B, C, and D) are abnormal findings, but do not indicate the development of septic emboli.

A client receiving cholestyramine (Questran) for hyperlipidemia should be evaluated for what vitamin deficiency? A) K. B) B12. C) B6. D) C.

A) K. Clients should be monitored for an increased prothrombin time and prolonged bleeding times which would alert the nurse to a vitamin K deficiency (A). These drugs reduce absorption of the fat soluble (lipid) vitamins A, D, E, and K. (B, C, and D) are not fat-soluble vitamins.

Small bowel obstruction is a condition characterized by which finding? A) Severe fluid and electrolyte imbalances. B) Metabolic acidosis. C) Ribbon-like stools. D) Intermittent lower abdominal cramping.

A) Severe fluid and electrolyte imbalances. Among the findings characteristic of a small bowel obstruction is the presence of severe fluid and electrolyte imbalances (A). (B, C, and D) are findings associated with large bowel obstruction.

A client who has heart failure is admitted with a serum potassium level of 2.9 mEq/L. Which action is most important for the nurse to implement? A) Give 20 mEq of potassium chloride. B) Initiate continuous cardiac monitoring. C) Arrange a consultation with the dietician. D) Teach about the side effects of diuretics.

B) Initiate continuous cardiac monitoring. Hypokalemia (normal 3.5 to 5 mEq/L) causes changes in myocardial irritability and ECG waveform, so it is most important for the nurse to initiate continuous cardiac monitoring (B) to identify ventricular ectopy or other life-threatening dysrhythmias. Potassium chloride (A) should be given after cardiac monitoring is initiated so that the effects of potassium replacement on the cardiac rhythm can be monitored. (C and D) should be implemented when the client is stable.

A 20-year-old female client calls the nurse to report a lump she found in her breast. Which response is the best for the nurse to provide? A) Check it again in one month, and if it is still there schedule an appointment. B) Most lumps are benign, but it is always best to come in for an examination. C) Try not to worry too much about it, because usually, most lumps are benign. D) If you are in your menstrual period it is not a good time to check for lumps

B) Most lumps are benign, but it is always best to come in for an examination. (B) provides the best response because it addresses the client's anxiety most effectively and encourages prompt and immediate action for a potential problem. (A) postpones treatment if the lump is malignant, and does not relieve the client's anxiety. (C and D) provide false reassurance and do not help relieve anxiety

Which intervention should the nurse plan to implement when caring for a client who has just undergone a right above-the-knee amputation? A) Maintain the residual limb on three pillows at all times. B) Place a large tourniquet at the client's bedside. C) Apply constant, direct pressure to the residual limb. D) Do not allow the client to lie in the prone position.

B) Place a large tourniquet at the client's bedside. A large tourniquet should be placed in plain sight at the client's bedside (B). If severe bleeding occurs, the tourniquet should be readily available and applied to the residual limb to control hemorrhage. The residual limb should not be placed on a pillow (A) because a flexion contracture of the hip may result. (C) should be avoided because it may compromise wound healing. (D) should be encouraged to stretch the flexor muscles and to prevent flexion contracture of the hip.

In assessing a client diagnosed with primary hyperaldosteronism, the nurse expects the laboratory test results to indicate a decreased serum level of which substance? A) Sodium. B) Antidiuretic hormone. C) Potassium. D) Glucose

C) Potassium. Clients with primary aldosteronism exhibit a profound decline in the serum levels of potassium (C) (hypokalemia)--hypertension is the most prominent and universal sign. (A) is normal or elevated, depending on the amount of water reabsorbed with the sodium. (B) is decreased with diabetes insipidus. (D) is not affected by primary aldosteronism.

A client with acute appendicitis is experiencing anxiety and loss of sleep about missing final examination week at college. Which outcome is most important for the nurse to include in the plan of care? A) Sleeping six to eight hours. B) Achieve a sense of control. C) Utilize problem solving skills. D) Increased focus of attention.

Correct Answer(s): B The experience of psychological discomfort may be as real as physical pain for the client and should be seen as a priority in care. Because the client is experiencing anxiety, achieving a sense of control is a key need (B) before (A,C and D) are addressed

A deficiency of intrinsic factor should alert the nurse to assess a client's history for which condition? A) Emphysema. B) Hemophilia. C) Pernicious anemia. D) Oxalic acid toxicity

Correct Answer(s): C Pernicious anemia (A) is a type of anemia due to failure of absorption of cobalamin (Vit B12). The most common cause is lack of intrinsic factor, a glucoprotein produced by the parietal cells of the gastric lining. (A, C, and D) are incorrect.

Which preexisting diagnosis places a client at greatest risk of developing superior vena cava syndrome? A) Carotid stenosis. B) Steatosis hepatitis. C) Metastatic cancer. D) Clavicular fracture.

Correct Answer(s): C Superior vena cava syndrome occurs when the superior vena cava (SVC) is compressed by outside structures, such as a growing tumor that impedes the return blood flow to the heart. Superior vena cava syndrome is likely to occur with metastatic cancer (C) from a primary tumor in the upper lobe of the right lung that compresses the superior vena cava. (A, B, and D) do not result in SVC syndrome.

The cardiac monitor of a 50-year-old client admitted for cocaine ingestion shows ventricular tachycardia (VT) converting to ventricular fibrillation (VF). What is the priority nursing action to implement? A) Prepare for intubation. B) Defibrillate at 200 joules. C) Insert intravenous catheter. D) Obtain arterial blood gases.

Correct Answer: B After confirming ventricular fibrillation, rapid defibrillation (B) is critical in re-establishing cardiac output and preserving vital organ function. After CPR is initiated and defibrillation attempted, airway intubation (A) and intravenous access (C) are indicated for successful resuscitation. Arterial blood gases (D) are obtained during or after resuscitation to determine medical management for metabolic acidosis which occurs secondary to anaerobic glycolysis during VF or cardiac arrest.

A client is receiving a continuous bladder irrigation at 1000 ml/hour after a prostatectomy. The nurse determines the client's urine output for the past hour is 200 ml. What action should the nurse implement first? A) Notify the healthcare provider. B) Stop the irrigation flow. C) Document the finding and continue to observe. D) Irrigate the catheter with a large piston syringe.

Correct Answer: B The urinary output should be at least the volume of irrigation input plus the client's actual urine. A significant decrease in output indicates obstruction in the drainage system, and the irrigation flow should be stopped (B) to prevent severe bladder distention. The next action is to check the external system for kinks or obstruction. If no output occurs, the catheter is irrigated with 30 to 50 ml of normal saline using a large iston syringe (D). If the obstruction is not resolved, then the healthcare provider (A) should be implemented.

A client with acute pancreatitis is admitted to the medical unit. During the nurse's admission interview, which assessment has the highest priority? A) History of alcohol intake. B) Time of last meal. C) Frequency of vomiting. D) Intensity of pain.

Correct Answer: D The hallmark sign of pancreatitis is severe abdominal pain (D), due to autodigestion of the pancreas by the enzymes amylase and lipase. (A, B, and C) are also important but are of less priority then (D).

The nurse working in the oncology clinic at a cancer center is involved in supporting clients and families who must cope with the diagnosis of cancer. Which client is likely to cope best with the diagnosis of cancer? A) An older man who is always happy and chooses to view only the good in every situation. B) A single mother who seeks the support of her two teenage daughters during difficult times. C) A successful businessman who is accustomed to handling highly-stressful situations. D) A teacher who seeks information about her disease and wants to continue teaching.

Correct Answer: D Those who seek information about their disease while attempting to carry on with their lives as best they can (D) are likely to handle the diagnosis of cancer best. Those who use repression (A) to deal with traumatic events often have difficulty expressing their feelings. Depending on children for support (B), especially when the children are teenagers, may be disappointing. Someone who is used to handling high-stress situations (C) is used to being in control, and control over a life-threatening diagnosis is not always possible.

The nurse is teaching a female client who uses a contraceptive diaphragm about reducing the risk for toxic shock syndrome (TSS). Which information should the nurse include? (Select all that apply.) A) Remove the diaphragm immediately after intercourse. B) Wash the diaphragm with an alcohol solution. C) Use the diaphragm to prevent conception during the menstrual cycle. D) Do not leave the diaphragm in place longer than 8 hours after intercourse. E) Contact a healthcare provider a sudden onset of fever greater than 101º F appears. F) Replace the old diaphragm every 3 months

Correct selections are (D and E). The diaphragm needs to remain against the cervix for 6 to 8 hours to prevent pregnancy but should not remain for longer than 8 hours (D) to avoid the risk of TSS. If a sudden fever occurs, the client should notify the healthcare provider (E). (A) increases the risk of pregnancy, and (B) can reduce the integrity of the barrier contraceptive but neither prevents the risk of TSS. The diaphragm should not be used during menses (C) because it obstructs the menstrual flow and is not indicated because conception does not occur during this time. (F) is not necessary.

When providing discharge teaching for a client with osteoporosis, the nurse should reinforce which home care activity? A) A diet low in phosphates. B) Skin inspection for bruising. C) Exercise regimen, including swimming. D) Elimination of hazards to home safety.

D) Elimination of hazards to home safety. Rationale: Discussion about fall prevention strategies is imperative for the discharged client with osteoporosis so that advice about safety measures can be given (D). A low phosphorus diet is not recommended in the treatment of osteoporosis (A). Bruising (B) is not a related symptom to osteoporosis. Weight-bearing exercise is most beneficial for clients with osteoporosis. Swimming (C) is not a weightbearing exercise.

A client with a completed ischemic stroke has a blood pressure of 180/90 mm Hg. Which action should the nurse implement? A) Position the head of the bed (HOB) flat. B) Withhold intravenous fluids. C) Administer a bolus of IV fluids. D) Give an antihypertensive medication

D) Give an antihypertensive medications. Most ischemic strokes occur during sleep when baseline blood pressure declines or blood viscosity increases due to minimal fluid intake. Completed strokes usually produce neurologic deficits within an hour, the client's current elevated blood pressure requires antihypertensive medication (D). Positioning the HOB flat (A) decreases venous drainage and contributes to cerebral edema post stroke. Increased blood viscosity during sleep may be related to reduced fluids, so (B) is not indicated. Increasing the vascular fluid volume increases the blood pressure, so (C) is not indicated

In preparing a discharge plan for a 22-year-old male client diagnosed with Buerger's disease (thromboangiitis obliterans), which referral is most important? A) Genetic counseling. B) Twelve-step recovery program. C) Clinical nutritionist. D) Smoking cessation program.

D) Smoking cessation program. Buerger's disease is strongly related to smoking. The most effective means of controlling symptoms and disease progression is through smoking cessation (D). The cause of Buerger's disease is unknown; a genetic predisposition is possible, but (A) will not be of value. The client with Buerger's disease does not need referral to a 12-step program any more than the general population (B). Diet is not a significant factor in the disease, and general healthy diet guidelines can be provided by the nurse (C).

n preparing to administer intravenous albumin to a client following surgery, what is the priority nursing intervention? (Select all that apply.) A) Set the infusion pump to infuse the albumin within four hours. B) Compare the client's blood type with the label on the albumin. C) Assign a UAP to monitor blood pressure q15 minutes. D) Administer through a large gauge catheter. E) Monitor hemoglobin and hematocrit levels. F) Assess for increased bleeding after administration.

(A, D, E, and F) are the correct selections. Albumin should be infused within four hours because it does not contain any preservatives. Any fluid remaining after four hours should be discarded (A). Albumin administration does not require blood typing (B). Vital signs should be monitored periodically to assess for fluid volume overload, but every 15 minutes is not necessary (C). This frequency is often used during the first hour of a blood transfusion. A large gauge catheter (D) allows for fast infusion rate, which may be necessary. Hemodilution may decrease hemoglobin and hematocrit levels (E), while increased blood volume and blood pressure may cause bleeding (F).

A client's susceptibility to ulcerative colitis is most likely due to which aspect in the client's history? A) Jewish European ancestry. B) H. pylori bowel infection. C) Family history of irritable bowel syndrome. D) Age between 25 and 55 years.

A Ulcerative colitis is 4 to 5 times more common among individuals of Jewish European or Ashkenazi ancestry (A). H. pylori is associated with stomach inflammation and ulcer development (B). Irritable bowel syndrome (C) does not progress to inflammatory bowel disease. UC has a peak between the ages of 15 and 25 years, then a second peak between 55 and 65 years, not (D).

An ER nurse is completing an assessment on a patient that is alert but struggles to answer questions. When she attempts to talk, she slurs her speech and appears very frightened. What additional clinical manifestation does the nurse expect to find if patient's symptoms have been caused by a brain attack (stroke)? A. A carotid bruit B. A hypotensive blood pressure C. hyperreflexic deep tendon reflexes. D. Decreased bowel sounds

A) A carotid bruit. Rationale: the carotid artery (artery to the brain) is narrowed in clients with a brain attack. A bruit is an abnormal sound heard on auscultation resulting from interference with normal blood flow. Usually the blood pressure is hypertensive. Initially flaccid paralysis occurs, resulting in hyporefkexic deep tendon reflexes. Bowel sounds are not indicative of a brain attack.

What instruction should the nurse include in the discharge teaching for a client who needs to perform self-catheterization technique at home? A) Catheterize every 3 to 4 hours. B) Maintain sterile technique. C) Use the Credé maneuver before catheterization. D) Drink 500 ml of fluid within 2 hours of catheterization

A) Catheterize every 3 to 4 hours. The average interval between catheterizations for adults is every 3 to 4 hours (A). Although sterile technique (B) is indicated in healthcare facilities, clean technique is often followed by the client when performing selfcatheterization at home. (C and D) are not indicated before self-catheterization.

A new nurse graduate is caring for a postoperative client with the following arterial blood gases (ABGs): pH, 7.30; PCO2, 60 mm Hg; PO2, 80 mm Hg; bicarbonate, 24 mEq/L; and O2 saturation, 96%. Which of these actions by the new graduate is indicated? A) Encourage the client to use the incentive spirometer and to cough. B) Administer oxygen by nasal cannula. C) Request a prescription for sodium bicarbonate from the health care provider. D) Inform the charge nurse that no changes in therapy are needed.

A) Encourage the client to use the incentive spirometer and to cough. Rationale: Respiratory acidosis is caused by CO2 retention and impaired chest expansion secondary to anesthesia. The nurse takes steps to promote CO2 elimination, including maintaining a patent airway and expanding the lungs through breathing techniques. O2 is not indicated because Po2 and oxygen saturation are within the normal range. Sodium bicarbonate is not indicated because the bicarbonate level is in the normal range; promoting excretion of respiratory acids is the priority in respiratory acidosis. Post anesthesia, the client will need interventions as described in A above or may progress to a state of somnolence and unresponsiveness.

A middle-aged male client with diabetes continues to eat an abundance of foods that are high in sugar and fat. According to the Health Belief Model, which event is most likely to increase the client's willingness to become compliant with the prescribed diet? A) He visits his diabetic brother who just had surgery to amputate an infected foot. B) He is provided with the most current information about the dangers of untreated diabetes. C) He comments on the community service announcements about preventing complications associated with diabetes. D) His wife expresses a sincere willingness to prepare meals that are within his prescribed diet

A) He visits his diabetic brother who just had surgery to amputate an infected foot. Rationale: The loss of a limb by a family member (A) will be the strongest event or "cue to action" and is most likely to increase the perceived seriousness of the disease. (B, C, and D) may influence his behavior but do not have the personal impact of (A).

A middle-aged male client with diabetes continues to eat an abundance of foods that are high in sugar and fat. According to the Health Belief Model, which event is most likely to increase the client's willingness to become compliant with the prescribed diet? A) He visits his diabetic brother who just had surgery to amputate an infected foot. B) He is provided with the most current information about the dangers of untreated diabetes. C) He comments on the community service announcements about preventing complications associated with diabetes. D) His wife expresses a sincere willingness to prepare meals that are within his prescribed diet.

A) He visits his diabetic brother who just had surgery to amputate an infected foot. The loss of a limb by a family member (A) will be the strongest event or "cue to action" and is most likely to increase the perceived seriousness of the disease. (B, C, and D) may influence his behavior but do not have the personal impact of (A).

An 81-year-old male client has emphysema. He lives at home with his cat and manages self-care with no difficulty. When making a home visit, the nurse notices that his tongue is somewhat cracked and his eyeballs are sunken into his head. What nursing intervention is indicated? A) Help the client to determine ways to increase his fluid intake. B) Obtain an appointment for the client to see an ear, nose, and throat specialist. C) Schedule an appointment with an allergist to determine if the client is allergic to the cat. D) Encourage the client to slightly increase his use of oxygen at night and to always use humidified oxygen.

A) Help the client to determine ways to increase his fluid intake. The nurse should suggest creative methods to increase the intake of fluids (A), such as having disposable fruit juices readily available. Clients with COPD should have at least three liters of fluids a day. These clients often reduce fluid intake because of shortness of breath. (B) is not indicated. These symptoms are not indicative of an allergy (C). Many elderly depend on their pets for socialization and selfesteem. Humidified oxygen will not relieve these symptoms and increased oxygen levels will stifle the COPD client's trigger to breathe (D).

A client is placed on a respirator following a cerebral hemorrhage, and vecuronium bromide (Norcuron) 0.04 mg/kg q12h IV is prescribed. Which nursing diagnosis is the priority for this client? A) Impaired communication related to paralysis of skeletal muscles. B) High risk for infection related to increased intracranial pressure. C) Potential for injury related to impaired lung expansion. D) Social isolation related to inability to communicate.

A) Impaired communication related to paralysis of skeletal muscles. To increase the client's tolerance of endotracheal intubation and/or mechanical ventilation, a skeletal-muscle relaxant such as vecuronium is usually prescribed. Impaired communication (A) is a serious outcome because the client cannot communicate his/her needs. Although this client might also experience (D), it is not a priority when compared to (A). Infection is not related to increased intracranial pressure (B). The respirator will ensure that the lungs are expanded (C).

A 51-year-old truck driver who smokes two packs of cigarettes a day and is 30 pounds overweight is diagnosed with having a gastric ulcer. What content is most important for the nurse to include in the discharge teaching for this client? A) Information about smoking cessation. B) Diet instructions for a low-residue diet. C) Instructions on a weight-loss program. D) The importance of increasing milk in the diet.

A) Information about smoking cessation. Smoking has been associated with ulcer formation, and stopping or decreasing the number of cigarettes smoked per day is an important aspect of ulcer management (A). Diet management includes a reduction in highfiber/high-roughage foods as well as spicy foods. (B) would be indicated for inflammatory bowel disease. Sodium and caloric intake are not the key elements in an ulcer diet. Although this client does need (C), the management of his ulcer is the key factor at this point. (D) would actually increase gastric acid production.

The nurse is caring for a client with syndrome of inappropriate antidiuretic hormone (SIADH), which is manifested by which symptoms? A) Loss of thirst, weight gain. B) Dependent edema, fever. C) Polydipsia, polyuria. D) Hypernatremia, tachypnea

A) Loss of thirst, weight gain. Rationale: SIADH occurs when the posterior pituitary gland releases too much ADH, causing water retention, a urine output of less than 20 ml/hour, and dilutional hyponatremia. Other indications of SIADH are loss of thirst, weight gain (A), irritability, muscle weakness, and decreased level of consciousness. (B) is not associated with SIADH. (C) is a finding associated with diabetes insipidus (a water metabolism problem caused by an ADH deficiency), not SIADH. The increase in plasma volume causes an increase in the glomerular filtration rate that inhibits the release of rennin and aldosterone, which results in an increased sodium loss in urine, leading to greater hyponatremia, not (D).

A client is admitted to the hospital with a diagnosis of severe acute diverticulitis. Which assessment finding should the nurse expect this client to exhibit? A) Lower left quadrant pain and a low-grade fever. B) Severe pain at McBurney's point and nausea. C) Abdominal pain and intermittent tenesmus. D) Exacerbations of severe diarrhea.

A) Lower left quadrant pain and a low-grade fever. Left lower quadrant pain occurs with diverticulitis because the sigmoid colon is the most common area for diverticula, and the inflammation of diverticula causes a low-grade fever (A). (B) would be indicative of appendicitis. (C and D) are symptoms exhibited with ulcerative colitis.

A client is admitted for further testing to confirm sarcoidosis. Which diagnostic test provides definitive information that the nurse should report to the healthcare provider? A) Lung tissue biopsy. B) Positive blood cultures. C) Magnetic resonance imaging (MRI). D) Computerized tomography (CT) of the thorax.

A) Lung tissue biopsy. Sarcoidosis is an inflammatory condition that is characterized by the formation of widespread granulomatous lesions involving a pulmonary primary site. Although chest radiography identifies sarcoidosis, lung tissue biopsy (A) obtained by bronchoscopy or bronchoalveolar lavage provides definitive confirmation. (B) does not provide results for sarcoidosis. Although MRI and CT identify pulmonary lesions, the (C and D) are not necessary and do not provide definitive confirmation

During the assessment of a client who is 24 hours posthemicolectomy with a temporary colostomy, the nurse determines that the client's stoma is dry and dark red in color. What action should the nurse implement? A) Notify the surgeon. B) Document the assessment. C) Secure a colostomy pouch over the stoma. D) Place petrolatum gauze dressing over the stoma.

A) Notify the surgeon. The stoma should appear reddish pink and moist, which indicates circulatory perfusion to the surgical diversion of the intestine. If the stoma becomes dry, firm, flaccid, or is dark red or purple, the stoma is ischemic, and the surgeon should be notified immediately (A). Although (B, C, and D) may be implemented, the findings requires immediate medical attention

When teaching diaphragmatic breathing to a client with chronic obstructive pulmonary disease (COPD), which information should the nurse provide? A) Place a small book or magazine on the abdomen and make it rise while inhaling deeply. B) Purse the lips while inhaling as deeply as possible and then exhale through the nose. C) Wrap a towel around the abdomen and push against the towel while forcefully exhaling. D) Place one hand on the chest, one hand the abdomen and make both hands move outward.

A) Place a small book or magazine on the abdomen and make it rise while inhaling deeply. Diaphragmatic or abdominal breathing uses the diaphragm instead of accessory muscles to achieve maximum inhalation and to slow the respiratory rate. The client should protrude the abdomen on inhalation and contract it with exhalation, so (A) helps the client visualize the rise and fall of the abdomen. The client should purse the lips while exhaling, not (B). (C and D) are ineffective.

The nurse is assessing a client with chronic renal failure (CRF). Which finding is most important for the nurse to respond to first? A) Potassium 6.0 mEq. B) Daily urine output of 400 ml. C) Peripheral neuropathy. D) Uremic fetor.

A) Potassium 6.0 mEq. Hyperkalemia (normal serum level, 3.5 to 5.5 mEq) is a serious electrolyte disorder that can cause fatal arrhythmias, so (A) is the nursing priority. (B) is an expected finding associated with renal tubular destruction. In CRF, an increase in serum nitrogenous waste products, electrolyte imbalances, and demyelination of the nerve fibers contribute to the development of (C). (D) is a urinous odor of the breath related to the accumulation of blood urea nitrogen and is a common complication of CRF, but not as significant as hyperkalemia.

The nurse should explain to a client with lung cancer that pleurodesis is performed to achieve which expected outcome? A) Prevent the formation of effusion fluid. B) Remove fluid from the intrapleural space. C) Debulk tumor to maintain patency of air passages. D) Relieve empyema after pneumonectomy.

A) Prevent the formation of effusion fluid. Instillation of a sclerosing agent to create pleurodesis (adherence of the parietal and visceral pleura) is aimed at preventing the formation of pleural effusion fluid (A). (B) refers to thoracentesis. (C) is achieved by surgical resection. (D) is treated by closed-chest drainage.

A splint is prescribed for nighttime use by a client with rheumatoid arthritis. Which statement by the nurse provides the most accurate explanation for use of the splints? A) Prevention of deformities. B) Avoidance of joint trauma. C) Relief of joint inflammation. D) Improvement in joint strength.

A) Prevention of deformities. Rationale: Splints may be used at night by clients with rheumatoid arthritis to prevent deformities (A) caused by muscle spasms and contractures. Splints are not used for (B). (C) is usually treated with medications, particularly those classified as non-steroidal antiinflammatory drugs (NSAIDs). For (D), a prescribed exercise program is indicated.

A client with heart disease is on a continuous telemetry monitor and has developed sinus bradycardia. In determining the possible cause of the bradycardia, the nurse assesses the client's medication record. Which medication is most likely the cause of the bradycardia? A) Propanolol (Inderal). B) Captopril (Capoten). C) Furosemide (Lasix). D) Dobutamine (Dobutrex)

A) Propanolol (Inderal). Inderal (A) is a beta-adrenergic blocking agent, which causes decreased heart rate and decreased contractility. Neither (B), an ACE inhibitor, nor (C), a loop diuretic, causes bradycardia. (D) is a sympathomimetic, direct acting cardiac stimulant, which would increase the heart rate.

A client with heart disease is on a continuous telemetry monitor and has developed sinus bradycardia. In determining the possible cause of the bradycardia, the nurse assesses the client's medication record. Which medication is most likely the cause of the bradycardia? A) Propanolol (Inderal). B) Captopril (Capoten). C) Furosemide (Lasix). D) Dobutamine (Dobutrex).

A) Propanolol (Inderal). Rationale: Inderal (A) is a beta adrenergic blocking agent, which causes decreased heart rate and decreased contractility. Neither (B), an ACE inhibitor, nor (C), a loop diuretic, causes bradycardia. (D) is a sympathomimetic, direct acting cardiac stimulant, which would increase the heart rate.

An adult client is admitted to the hospital burn unit with partial-thickness and full-thickness burns over 40% of the body surface area. In assessing the potential for skin regeneration, what should the nurse remember about fullthickness burns? A) Regenerative function of the skin is absent because the dermal layer has been destroyed. B) Tissue regeneration will begin several days following return of normal circulation. C) Debridement of eschar will delay the body's ability to regenerate normal tissue. D) Normal tissue formation will be preceded by scar formation for the first year

A) Regenerative function of the skin is absent because the dermal layer has been destroyed. Full-thickness burns destroy the entire dermal layer. Included in this destruction is the regenerative tissue. For this reason, tissue regeneration does not occur, and skin grafting is necessary (A). (B, C, and D) are simply false.

After checking the urinary drainage system for kinks in the tubing, the nurse determines that a client who has returned from the post-anesthesia care has a dark, concentrated urinary output of 54 ml for the last 2 hours. What priority nursing action should be implemented? A) Report the findings to the surgeon. B) Irrigate the indwelling urinary catheter. C) Apply manual pressure to the bladder. D) Increase the IV flow rate for 15 minutes.

A) Report the findings to the surgeon. An adult who weighs 132 pounds (60 kg) should produce about 60 ml of urine hourly (1 ml/kg/hour). Dark, concentrated, and low volume of urine output should be reported to the surgeon. Although other actions (B, C, and D) may be indicated, the assessment findings should be reported to the healthcare provider

The nurse working in a postoperative surgical clinic is assessing a woman who had a left radical mastectomy for breast cancer. Which factor puts this client at greatest risk for developing lymphedema? A) She sustained an insect bite to her left arm yesterday. B) She has lost twenty pounds since the surgery. C) Her healthcare provider now prescribes a calcium channel blocker for hypertension. D) Her hobby is playing classical music on the piano.

A) She sustained an insect bite to her left arm yesterday. A radical mastectomy interrupts lymph flow, and the increased lymph flow that occurs in response to the insect bite increases the risk for the occurrence of lymphedema (A). (B) is not a factor. Lymphedema is not significantly related to vascular circulation (C). Only overuse of the arm, such as weight-lifting, would cause lymphedema--(D) would not.

A female client receiving IV vasopressin (Pitressin) for esophageal varice rupture reports to the nurse that she feels substernal tightness and pressure across her chest. Which PRN protocol should the nurse initiate? A) Start an IV nitroglycerin infusion. B) Nasogastric lavage with cool saline. C) Increase the vasopressin infusion. D) Prepare for endotracheal intubation.

A) Start an IV nitroglycerin infusion. Rationale: Vasopressin is used to promote vasoconstriction, thereby reducing bleeding. Vasoconstriction of the coronary arteries can lead to angina and myocardial infarction, and should be counteracted by IV nitroglycerin per prescribed protocol (A). (B) will not resolve the cardiac problem. (C) will worsen the problem. Endotracheal intubation may be needed if respiratory distress occurs (D)

The nurse is preparing a teaching plan for a client who is newly diagnosed with Type 1 diabetes mellitus. Which signs and symptoms should the nurse describe when teaching the client about hypoglycemia? A) Sweating, trembling, tachycardia. B) Polyuria, polydipsia, polyphagia. C) Nausea, vomiting, anorexia. D) Fruity breath, tachypnea, chest pain.

A) Sweating, trembling, tachycardia. Rationale: Sweating, dizziness, and trembling are signs of hypoglycemic reactions related to the release of epinephrine as a compensatory response to the low blood sugar (A). (B, C, and D) do not describe common symptoms of hypoglycemia.

What assessment finding should the nurse identify that indicates a client with an acute asthma exacerbation is beginning to improve after treatment? A) Wheezing becomes louder. B) Cough remains unproductive. C) Vesicular breath sounds decrease. D) Bronchodilators stimulate coughing.

A) Wheezing becomes louder. In an acute asthma attack, air flow may be so significantly restricted that wheezing is diminished. If the client is successfully responding to bronchodilators and respiratory treatments, wheezing becomes louder (A) as air flow increases in the airways. As the airways open and mucous is mobilized in response to treatment, the cough becomes more productive, not (B). Vesicular sounds are soft, low-pitched, gentle, rustling sounds heard over lung fields (C) and is not an indicator of improvement during asthma treatment. Bronchodilators do not stimulate coughing (D).

During suctioning, a client with an uncuffed tracheostomy tube begins to cough violently and dislodges the tracheostomy tube. Which action should the nurse implement first? A) Notify the healthcare provider for reinsertion. B) Attempt to reinsert the tracheostomy tube. C) Position the client in a lateral position with the neck extended. D) Ventilate client's tracheostomy stoma with a manual bag-mask.

B) Attempt to reinsert the tracheostomy tube. The nurse should attempt to reinsert the tracheostomy tube (B) by using a hemostat to open the tracheostomy or by grasping the retention sutures (if present) to spread the opening in insert a replacement tube (with its obturator) into the stoma. Once in place, the obturator should immediately be removed. (A, C, and D) place the client at risk of airway obstruction.

The nurse completes visual inspection of a client's abdomen. What technique should the nurse perform next in the abdominal examination? A) Percussion. B) Auscultation. C) Deep palpation. D) Light palpation.

B) Auscultation. Auscultation (B) of the client's abdomen is performed next because manual manipulation (A, C, and D) can stimulate the bowel and create false sounds heard during auscultation.

A 58-year-old client who has been post-menopausal for five years is concerned about the risk for osteoporosis because her mother has the condition. Which information should the nurse offer? A) Osteoporosis is a progressive genetic disease with no effective treatment. B) Calcium loss from bones can be slowed by increasing calcium intake and exercise. C) Estrogen replacement therapy should be started to prevent the progression osteoporosis. D) Low-dose corticosteroid treatment effectively halts the course of osteoporosis.

B) Calcium loss from bones can be slowed by increasing calcium intake and exercise. Post-menopausal females are at risk for osteoporosis due to the cessation of estrogen secretion, but a regimen including calcium, vitamin D, and weight-bearing exercise can prevent further bone loss (B). Osteoporosis can be managed with conservative therapy, such as bone metabolism regulators and estrogen replacement therapy (ERT) to improve bone density, but it is not a genetic disease (A). Although ERT is effective in managing osteoporosis, an increased risk for cancer and heart disease should be considered for individual clients. Corticosteroid therapy promotes bone resorption and is counterproductive in maintaining or increasing bone density (D).

The nurse notes that the only ECG for a 55-year-old male client scheduled for surgery in two hours is dated two years ago. The client reports that he has a history of "heart trouble," but has no problems at present. Hospital protocol requires that those over 50 years of age have a recent ECG prior to surgery. What nursing action is best for the nurse to implement? A) Ask the client what he means by "heart trouble." B) Call for an ECG to be performed immediately. C) Notify surgery that the ECG is over two years old. D) Notify the client's surgeon immediately.

B) Call for an ECG to be performed immediately. Clients over the age of 40 and/or with a history of cardiovascular disease, should receive ECG evaluation prior to surgery, generally 24 hours to two weeks before. (B) should be implemented to ensure that the client's current cardiovascular status is stable. Additional data might be valuable (A), but since time is limited, the priority is to obtain the needed ECG. Documentation of vital signs is important, but does not replace the need for the ECG (C). The surgeon only needs to be notified if the ECG cannot be completed, or if there is a significant problem (D).

The nurse is caring for a client with a continuous feeding through a percutaneous endoscopic gastrostomy (PEG) tube. Which intervention should the nurse include in the plan of care? A) Flush the tube with 50 ml of water q 8 hours. B) Check for tube placement and residual volume q4 hours. C) Obtain a daily x- ray to verify tube placement. D) Position on left side with head of bed elevated 45 degrees.

B) Check for tube placement and residual volume q4 hours. Tube placement and residual volume should be checked before each feeding (B). Tube placement is checked by aspiration of stomach contents and measurement of pH. It is important to check for residual volume because gastric emptying is often delayed during illness. There is an increased risk for aspiration of the feeding with increased residual volume. (A, C, and D) are not correct procedures to follow.

The nurse is caring for a client with non-Hodgkin's lymphoma who is receiving chemotherapy. Laboratory results reveal a platelet count of 10,000/ml. What action should the nurse implement? A) Encourage fluids to 3000 ml/day. B) Check stools for occult blood. C) Provide oral hygiene every 2 hours. D) Check for fever every 4 hours.

B) Check stools for occult blood. Platelet counts less than 100,000/mm3 are indicative of thrombocytopenia, a common side effect of chemotherapy. A client with thrombocytopenia should be assessed frequently for occult bleeding in the emesis, sputum, feces (B), urine, nasogastric secretions, or wounds. (A) does not minimize the risk for bleeding associated with thrombocytopenia. (C) may cause increased bleeding in a client with thromobcytopenia. (D) assesses for infection, not risk for bleeding.

The nurse hears short, high-pitched sounds just before the end of inspiration in the right and left lower lobes when auscultating a client's lungs. How should this finding be recorded? A) Inspiratory wheezes in both lungs. B) Crackles in the right and left lower lobes. C) Abnormal lung sounds in the bases of both lungs. D) Pleural friction rub in the right and left lower lobes.

B) Crackles in the right and left lower lobes. Fine crackles (B) are short, high-pitched sounds heard just before the end of inspiration that are the result of rapid equalization of pressure when collapsed alveoli or terminal bronchioles suddenly snap open. Wheezing (A) is a continuous high-pitched squeaking or musical sound caused by rapid vibration of bronchial walls that are first evident on expiration and may be audible. Although (C) describes an adventitious lung sound, this documentation is vague. (D) is a creaking or grating sound from roughened, inflamed surfaces of the pleura rubbing together heard during inspiration, expiration, and with no change during coughing.

The nurse is caring for a client with a stroke resulting in right-sided paresis and aphasia. The client attempts to use the left hand for feeding and other self-care activities. The spouse becomes frustrated and insists on doing everything for the client. Based on this data, which nursing diagnosis should the nurse document for this client? A) Situational low self-esteem related to functional impairment and change in role function. B) Disabled family coping related to dissonant coping style of significant person. C) Interrupted family processes related to shift in health status of family member. D) Risk for ineffective therapeutic regimen management related to complexity of care.

B) Disabled family coping related to dissonant coping style of significant person. A stroke affects the whole family and in this case the spouse probably thinks that she is helping and needs to feel that she is contributing to the client's care. Her help is noted as being incongruent with attempts of self-care by the client thereby disabling family coping (B). The scenario does not discuss the client's self-esteem (A), interrupted family processes (C) or the risk for ineffective therapeutic regimen (D).

A 49-year-old female client arrives at the clinic for an annual exam and asks the nurse why she becomes excessively diaphoretic and feels warm during nighttime. What is the nurse's best response? A) Explain the effect of the follicle-stimulating and luteinizing hormones. B) Discuss perimenopause and related comfort measures. C) Assess lung fields and for a cough productive of bloodtinged mucous. D) Ask if a fever above 101º F has occurred in the last 24 hours.

B) Discuss perimenopause and related comfort measures. The perimenopausal period begins about 10 years before menopause with the cessation of menstruation at the average ages of 52 to 54. Lower estrogen levels cause FSH and LH secretion in bursts (surges), which triggers vasomotor instability, night sweats, and hot flashes, so discussions about the perimenopausal body's changes, comfort measures (B), and treatment options should be provided. In-depth pathophysiology of the symptoms (A) may only confuse the client. There is no indication that the client has tuberculosis and an infection, so (C and D) are not indicated.

A male client receives a local anesthetic during surgery. During the post-operative assessment, the nurse notices the client is slurring his speech. Which action should the nurse take? A) Determine the client is anxious and allow him to sleep. B) Evaluate his blood pressure, pulse, and respiratory status. C) Review the client's pre-operative history for alcohol abuse. D) Continue to monitor the client for reactivity to anesthesia.

B) Evaluate his blood pressure, pulse, and respiratory status. Rationale: Slurred speech in the post-operative client who received a local anesthetic is an atypical finding and may indicate neurological deficits that require further assessment, so obtaining the client's vital signs (B) will provide information about possible cardiovascular complications, such as stroke. The client's anxiety (A), a history of alcohol abuse (D), or local anesthesia (D) are unrelated to the client's sudden onset of slurred speech.

When preparing a patient for a noncontrast computed tomography (CT) scan STAT, what nursing intervention should the nurse implement? A) Determine if the client has any allergies to iodine B) Explain that the client will not be able to move her head throughout the CT scan. C) Pre-medicate the client to decrease pain prior to having the procedure. D) Provide an explanation of relaxation exercises prior to the procedure.

B) Explain that the client will not be able to move her head throughout the CT scan. Rationale: Because head motion will distort the images, Nancy will have to remain still throughout the procedure. Allergies to iodine is important if contrast dye is being used for the CT scan. Pre-medicating the client to decrease pain prior to the procedure is unnecessary because CT scanning is a noninvasive and painless procedure. Providing an explanation of relaxation exercises prior to the procedure is a worthwhile intervention to decrease anxiety but is not of highest priority

The nurse is caring for a client with end stage liver disease who is being assessed for the presence of asterixis. To assess the client for asterixis, what position should the nurse ask the client to demonstrate? A) Extend the left arm laterally with the left palm upward. B) Extend the arm, dorsiflex the wrist, and extend the fingers. C) Extend the arms and hold this position for 30 seconds. D) Extend arms with both legs adducted to shoulder width.

B) Extend the arm, dorsiflex the wrist, and extend the fingers. Asterixis (flapping tremor, liver flap) is a hand-flapping tremor that is often seen frequently in hepatic encephalopathy. The tremor is induced by extending the arm and dorsiflexing the wrist causing rapid, nonrhythmic extension and flexion of the wrist while attempting to hold position (B). (A, C, and D) do not illicit asterixis.

The nurse is caring for a client who has taken a large quantity of furosemide (Lasix) to promote weight loss. The nurse anticipates the finding of which acid-base imbalance? A) PO2 of 78 mm Hg B) HCO3 of 34 mEq/L C) PCO2 of 56 mm Hg D) pH of 7.31

B) HCO3 of 34 mEq/L Rationale: Diuretics (non-potassium sparing) cause metabolic alkalosis. A) PO2 of 78 mm Hg: This Po2 demonstrates mild hypoxemia, consistent with respiratory disorders, not with diuretic use. C) PCO2 of 56 mm Hg: CO2 retention results from hypoventilation, which is not consistent with diuretic use. D) pH of 7.31: This pH is acidotic; diuretics promote metabolic alkalosis.

Which symptoms should the nurse expect a client to exhibit who is known to have a pheochromocytoma? A) Numbness, tingling, and cramps in the extremities. B) Headache, diaphoresis, and palpitations. C) Cyanosis, fever, and classic signs of shock. D) Nausea, vomiting, and muscular weakness.

B) Headache, diaphoresis, and palpitations. (B) is the typical triad of symptoms of tumors of the adrenal medulla (symptoms depend on the relative proportions of epinephrine and norepinephrine secretion). (A) lists the signs of latent tetany, exhibited by clients diagnosed with hypoparathyroidism. (C) lists the signs of an Addisonian (adrenal) crisis. (D) lists the signs of hyperparathyroidism.

The nurse is providing dietary instructions to a 68-yearold client who is at high risk for development of coronary heart disease (CHD). Which information should the nurse include? A) Limit dietary selection of cholesterol to 300 mg per day B) Increase intake of soluble fiber to 10 to 25 grams per day. C) Decrease plant stanols and sterols to less than 2 grams/day. D) Ensure saturated fat is less than 30% of total caloric intake

B) Increase intake of soluble fiber to 10 to 25 grams per day. Rationale: To reduce risk factors associated with coronary heart disease, the daily intake of soluble fiber (B) should be increased to between 10 and 25 gm. Cholesterol intake (A) should be limited to 180 mg/day or less. Intake of plant stanols and sterols is recommended at 2 g/day (C). Saturated fat (D) intake should be limited to 7% of total daily calories.

Which reaction should the nurse identify in a client who is responding to stimulation of the sympathetic nervous system? A) Pupil constriction. B) Increased heart rate. C) Bronchial constriction. D) Decreased blood pressure.

B) Increased heart rate. Rationale: Any stressor that is perceived as threatening to homeostasis acts to stimulate the sympathetic nervous system and manifests as a flight-or-fight response, which includes an increase in heart rate (B). (A, C, and D) are responses of the parasympathetic nervous system.

A client with a 16-year history of diabetes mellitus is having renal function tests because of recent fatigue, weakness, elevated blood urea nitrogen, and serum creatinine levels. Which finding should the nurse conclude as an early symptom of renal insufficiency? A) Dyspnea. B) Nocturia. C) Confusion. D) Stomatitis.

B) Nocturia. Rationale: As the glomerular filtration rate decreases in early renal insufficiency, metabolic waste products, including urea, creatinine, and other substances, such phenols, hormones, electrolytes, accumulate in the blood. In the early stage of renal insufficiency, polyuria results from the inability of the kidneys to concentrate urine and contribute to nocturia (B). (A, C, and D) are more common in the later stages of renal failure

Which client should the nurse recognize as most likely to experience sleep apnea? A) Middle-aged female who takes a diuretic nightly. B) Obese older male client with a short, thick neck. C) Adolescent female with a history of tonsillectomy. D) School-aged male with a history of hyperactivity disorder

B) Obese older male client with a short, thick neck. Rationale: Sleep apnea is characterized by lack of respirations for 10 seconds or more during sleep and is due to the loss of pharyngeal tone which allows the pharynx to collapse during inspiration and obstructs air flow through the nose and mouth. With obstructive sleep apnea, the client is often obese or has a short, thick neck as in (B). (A, C, and D) are not typically prone to sleep apnea.

A client is experiencing homonymous hemianopsia as the result of a brain attack. Which nursing intervention would the nurse implement to address this condition? A) Turn Nancy every two hours and perform active range of motion exercises. B) Place the objects Nancy needs for activities of daily living on the left side of the table. C) Speak slowly and clearly to assist Nancy in forming sounds to words. D) Request that the dietary department thicken all liquids on Nancy's meal and snack trays.

B) Place the objects Nancy needs for activities of daily living on the left side of the table. Rationale: Homonymous hemianopsia is loss of the visual field on the same side as the paralyzed side. This results in the client neglecting that side of the body, so it is beneficial to place objects on that side. Nancy had a left-hemisphere brain attack so her right side is the weak side. Speaking slowly and clearly would address the client's verbal deficits due to aphasia. Requesting all liquids to be thickened would address dysphagia. Turning the client every 2 hours and performing active range of motion exercises would address the client's risk for immobility due to paralysis.

The nurse is preparing discharge instructions for a client who is going home with a surgical wound on the coccyx that is healing by second intention. What is the priority nursing diagnosis that should guide the discharge instruction plan? A) Acute pain. B) Risk for infection. C) Disturbed body image. D) Risk for deficient fluid volume

B) Risk for infection. A wound healing by second intention is an open wound that is at risk for infection (B). Discomfort should be minimal 2 days after surgery, and acute pain (A) is not the priority. Risk for deficient fluid volume (D) requires a significant amount of wound draining, which is not evident. Although a wound may contribute to a disturbed body image (C), the client's distress may be minimal because the wound is not visible to others.

The nurse is planning care for a client who has a right hemispheric stroke. Which nursing diagnosis should the nurse include in the plan of care? A) Impaired physical mobility related to right-sided hemiplegia. B) Risk for injury related to denial of deficits and impulsiveness. C) Impaired verbal communication related to speechlanguage deficits. D) Ineffective coping related to depression and distress about disability.

B) Risk for injury related to denial of deficits and impulsiveness. With right-brain damage, a client experience difficulty in judgment and spatial perception and is more likely to be impulsive and move quickly, which placing the client at risk for falls (B). Although clients with right and left hemisphere damage may experience impaired physical mobility, the client with right brain damage will manifest physical impairments on the contralateral side of the body, not the same side (A). The client with a left-brain injury may manifest right-sided hemiplegia with speech or language deficits (C). A client with left-brain damage is more likely to be aware of the deficits and experience grief related to physical impairment and depression (D)

A client is admitted to the hospital with a traumatic brain injury after his head violently struck a brick wall during a gang fight. Which finding is most important for the nurse to assess further? A) A scalp laceration oozing blood. B) Serosanguineous nasal drainage. C) Headache rated 10 on a 0-10 scale. D) Dizziness, nausea and transient confusion.

B) Serosanguineous nasal drainage. Any nasal discharge should be evaluated (B) to determine the presence of cerebral spinal fluid which indicates a tear in the dura making the client susceptible to meningitis. The scalp is highly vascular and results in blood oozing from wounds (A). Pain is expected and can be treated after further assessment of the presence of nasal discharge (C). Dizziness, nausea, and transient confusion (D) are expected manifestations following a traumatic brain injury and need ongoing monitoring, but (B) is most important.

Physical examination of a comatose client reveals decorticate posturing. Which statement is accurate regarding this client's status based upon this finding? A) A cerebral infectious process is causing the posturing. B) Severe dysfunction of the cerebral cortex has occurred. C) There is a probable dysfunction of the midbrain. D) The client is exhibiting signs of a brain tumor.

B) Severe dysfunction of the cerebral cortex has occurred. Decorticate posturing (adduction of arms at shoulders, flexion of arms on chest with wrists flexed and hands fisted and extension and adduction of extremities) is seen with severe dysfunction of the cerebral cortex (B). (A) is characteristic of meningitis. (C) is characterized by decerebrate posturing (rigid extension and pronation of arms and legs). A client with (D) may exhibit decorticate posturing, depending on the position of the tumor and the condition of the client

The nurse working on a telemetry unit finds a client unconscious and in pulseless ventricular tachycardia (VT). The client has an implanted automatic defibrillator. What action should the nurse implement? A) Prepare the client for transcutaneous pacemaker. B) Shock the client with 200 joules per hospital policy. C) Use a magnet to deactivate the implanted pacemaker. D) Observe the monitor until the onset of ventricular fibrillation.

B) Shock the client with 200 joules per hospital policy. The client must be externally shocked (B) to restore an effective cardiac rhythm. The automatic defibrillator is obviously malfunctioning. (A) will not be effective during ventricular tachycardia, since it is used for asystole. Since the defibrillator is not functioning, (C) is not warranted. The client should be treated immediately to restore cardiac output (D).

Which description of symptoms is characteristic of a client diagnosed with trigeminal neuralgia (tic douloureux)? A) Tinnitus, vertigo, and hearing difficulties. B) Sudden, stabbing, severe pain over the lip and chin. C) Facial weakness and paralysis. D) Difficulty in chewing, talking, and swallowing.

B) Sudden, stabbing, severe pain over the lip and chin. Trigeminal neuralgia is characterized by paroxysms of pain, similar to an electric shock, in the area innervated by one or more branches of the trigeminal nerve (5th cranial) (B). (A) would be characteristic of Méniére's disease (8th cranial nerve). (C) would be characteristic of Bell's palsy (7th cranial nerve). (D) would be characteristic of disorders of the hypoglossal cranial nerve (12th).

A 58-year-old client, who has no health problems, asks the nurse about the Pneumovax vaccine. The nurse's response to the client should be based on which information? A) The vaccine is given annually before the flu season to those over 50 years of age. B) The immunization is administered once to older adults or persons with a history of chronic illness. C) The vaccine is for all ages and is given primarily to those persons traveling overseas to areas of infection. D) The vaccine will prevent the occurrence of pneumococcal pneumonia for up to five years.

B) The immunization is administered once to older adults or persons with a history of chronic illness. It is usually recommended that persons over 65 years of age and those with a history of chronic illness receive the vaccine once in a lifetime (B). (Some resources recommend obtaining the vaccine at 50 years of age.) The influenza vaccine is given once a year, not the Pneumovax (A). Although the vaccine might be given to a person traveling overseas, that is not the main rationale for administering the vaccine (C). It is usually given once in a lifetime (D), but with immunosuppressed clients or clients with a history of pneumonia re-vaccination is sometimes required.

Healthcare workers must protect themselves against becoming infected with HIV. The Center for Disease Control has issued guidelines for healthcare workers in relation to protection from HIV. These guidelines include which recommendation? A) Place HIV positive clients in strict isolation and limit visitors. B) Wear gloves when coming in contact with the blood or body fluids of any client. C) Conduct mandatory HIV testing of those who work with AIDS clients. D) Freeze HIV blood specimens at -70° F to kill the virus

B) Wear gloves when coming in contact with the blood or body fluids of any client. The CDC guidelines recommend that healthcare workers use gloves when coming in contact with blood or body fluids from ANY client (B) since HIV is infectious before the client becomes aware of symptoms. (A) is not recommended, nor is it necessary. (C) is very controversial, difficult to enforce, and is not recommended by CDC. (D) does not guarantee to kill the virus. Additionally, the purpose of the blood specimen will determine how it is stored and handled.

A client is admitted to the hospital with a medical diagnosis of pneumococcal pneumonia. The nurse knows that the prognosis for gram-negative pneumonias (such as E. coli, Klebsiella, Pseudomonas, and Proteus) is very poor because A) they occur in the lower lobe alveoli which are more sensitive to infection. B) gram-negative organisms are more resistant to antibiotic therapy. C) they occur in healthy young adults who have recently been debilitated by an upper respiratory infection. D) gram-negative pneumonias usually affect infants and small children

B) gram-negative organisms are more resistant to antibiotic therapy. The gram-negative organisms are resistant to drug therapy (B) which makes recovery very difficult. Gramnegative pneumonias affect all lobes of the lung (A). The mean age for contracting this type of pneumonia is 50 years (C and D), and it usually strikes debilitated persons such as alcoholics, diabetics, and those with chronic lung diseases.

Which finding should the nurse identify as an indication of carbon monoxide poisoning in a client who experienced a burn injury during a house fire? A) Pulse oximetry reading of 80%. B) Expiratory stridor and nasal flaring. C) Cherry red color to the mucous membranes. D) Presence of carbonaceous particles in sputum.

C The saturation of hemoglobin molecules with carbon monoxide and the subsequent vasodilation induce a cherry red color of the mucous membranes (C) in a client who experienced a burn injury during a house fire. Super heated air or smoke inhalation damage the lining of the airways which causes swelling, decreased oxygenation (A), and an expiratory stridor (B). Mouth breathing during the fire allows the inhalation of soot that is seen as particles in the client's sputum (D).

Which finding should the nurse identify as most significant for a client diagnosed with polycystic kidney disease (PKD)? A) Hematuria. B) 2 pounds weight gain. C) 3+ bacteria in urine. D) Steady, dull flank pain.

C) 3+ bacteria in urine. Urinary tract infections (UTI) for a client with PKD require prompt antibiotic therapy to prevent renal damage and scarring which may cause further progression of the disease, so bacteria in the urine (C) is the most significant finding at this time. (A) is an expected finding from the rupture of the cysts. (B) does not provide a time frame to determine if the weight gain is a significant fluid fluctuation, which is determined within a 24-hour time frame. Although kidney pain can also be abrupt, episodic, and colicky related to bleeding into the cysts, (D) is more likely an early symptom in PKD

After the fourth dose of gentamicin sulfate (Garamycin) IV, the nurse plans to draw blood samples to determine peak and trough levels. When are the best times to draw these samples? A) 15 minutes before and 15 minutes after the next dose. B) One hour before and one hour after the next dose. C) 5 minutes before and 30 minutes after the next dose. D) 30 minutes before and 30 minutes after the next dose.

C) 5 minutes before and 30 minutes after the next dose. Rationale: Peak drug serum levels are achieved 30 minutes after IV administration of aminoglycosides. The best time to draw a trough is the closest time to the next administration (C). (A, B, and D) are not as good a time to draw the trough as (C). (B and D) are not the best times to draw the peak of an aminoglycoside that has been administered IV.

A client taking a thiazide diuretic for the past six months has a serum potassium level of 3. The nurse anticipates which change in prescription for the client? A) The dosage of the diuretic will be decreased. B) The diuretic will be discontinued. C) A potassium supplement will be prescribed. D) The dosage of the diuretic will be increased.

C) A potassium supplement will be prescribed. This client's potassium level is too low (normal is 3.5 to 5). Taking a thiazide diuretic often results in a loss of potassium, so a potassium supplement needs to be prescribed to restore a normal serum potassium level (C). (A, B, and D) are not recommended actions for restoring a normal serum potassium level.

A female client taking oral contraceptives reports to the nurse that she is experiencing calf pain. What action should the nurse implement? A) Determine if the client has also experienced breast tenderness and weight gain. B) Encourage the client to begin a regular, daily program of walking and exercise. C) Advise the client to notify the healthcare provider for immediate medical attention. D) Tell the client to stop taking the medication for a week to see if symptoms subside.

C) Advise the client to notify the healthcare provider for immediate medical attention. Rationale: Calf pain is indicative of thrombophlebitis, a serious, life-threatening complication associated with the use of oral contraceptives which requires further assessment and possibly immediate medical intervention (C). (A) are symptoms of oral contraceptive use, but are of less immediacy than (C). (B) may cause an embolism if thrombophlebitis is present. By not seeking immediate attention, (D) is potentially dangerous to the client.

Based on the analysis of the client's atrial fibrillation, the nurse should prepare the client for which treatment protocol? A) Diuretic therapy. B) Pacemaker implantation. C) Anticoagulation therapy. D) Cardiac catheterization.

C) Anticoagulation therapy. The client is experiencing atrial fibrillation, and the nurse should prepare the client for anticoagulation therapy (C) which should be prescribed before rhythm control therapies to prevent cardioembolic events which result from blood pooling in the fibrillating atria. (A, B, and D) are not indicated.

The nurse is caring for a group of clients with acidosis. The nurse recognizes that Kussmaul respirations are consistent with which situation? A) Client receiving mechanical ventilation B) Use of hydrochlorothiazide C) Aspirin overdose D) Administration of sodium bicarbonate

C) Aspirin overdose Rationale: If acidosis is metabolic in origin, the rate and depth of breathing increase as the hydrogen ion level rises; this is known as Kussmaul respirations. Metabolic acidosis is caused by alcoholic beverages, methyl alcohol, and acetylsalicylic acid (aspirin). A) Mechanical ventilation is used to correct hypoxemia and hypercapnia (elevated Pco2). B) Hydrochlorothiazide causes metabolic alkalosis; clients who display metabolic acidosis compensate with Kussmaul respirations. D) Sodium bicarbonate is used in the treatment of metabolic acidosis; administration of this buffer may cause metabolic alkalosis.

The nurse is preparing a teaching plan for a client with newly diagnosed glaucoma and a history of allergic rhinitis. Which information is most important for the nurse to provide the client about using over-the-counter (OTC) medications for allergies? A) Notify your healthcare provider if there is an increase in heart rate. B) Increase fluid intake while taking an antihistamine or decongestant. C) Avoid allergy medications that contain pseudoephedrine or phenylephrine. D) Ophthalmic lubricating drops may be used for eye dryness due to allergy medications.

C) Avoid allergy medications that contain pseudoephedrine or phenylephrine. OTC allergy medications may contain ephedrine, phenylephrine, or pseudoephedrine, which can cause adrenergic side effects, such as increased intraocular pressure, so a client with glaucoma should avoid using these OTC medications (C). A client with hypertension should avoid using OTC medications containing ingredients that can increase blood pressure and heart rate (A), but an increase in IOP is most important in a client with glaucoma. (B and D) may provide symptomatic relief for other side effects, such as dry mouth or eye dryness related to common agents used for allergic rhinitis.

The nurse is teaching a client with maple syrup urine disease (MSUD), an autosomal recessive disorder, about the inheritance pattern. Which information should the nurse provide? A) This recessive disorder is carried only on the X chromosome. B) Occurrences mainly affect males and heterozygous females. C) Both genes of a pair must be abnormal for the disorder to occur. D) One copy of the abnormal gene is required for this disorder

C) Both genes of a pair must be abnormal for the disorder to occur. Maple syrup urine disease (MSUD) is a type of autosomal recessive inheritance disorder in which both genes of a pair must be abnormal for the disorder to be expressed (C). MSUD is not an x-linked (A and B) dominant or recessive disorder or an autosomal dominant inheritance disorder. Both genes of a pair, not (D), must be present.

A client is admitted to the medical intensive care unit with a diagnosis of myocardial infarction. The client's history indicates the infarction occurred ten hours ago. Which laboratory test result should the nurse expect this client to exhibit? A) Elevated LDH. B) Elevated serum amylase. C) Elevated CK-MB. D) Elevated hematocrit.

C) Elevated CK-MB. The cardiac isoenzyme CK-MB (C) is the most sensitive and most reliable indicator of myocardial damage of all the cardiac enzymes. It peaks within 12 to 20 hours after myocardial infarction (MI). (A) is a cardiac enzyme that peaks around 48 hours after an MI. (B) is expected with acute pancreatitis. (D) would be expected in a client with a fluid volume deficit, which is not a typical finding in MI.

A female client with type 2 diabetes mellitus reports dysuria. Which assessment finding is most important for the nurse to report to the healthcare provider? A) Suprapubic pain and distention. B) Bounding pulse at 100 beats/minute. C) Fingerstick glucose of 300 mg/dl. D) Small vesicular perineal lesions.

C) Fingerstick glucose of 300 mg/dl. Elevated fingerstick glucose levels (C) spill glucose in the urine and provide a medium for bacterial growth. (A, B, and D) should be reported, but the priority (C) is to notify the healthcare provider for prescriptions to manage client to a euglycemic level.

While working in the emergency room, the nurse is exposed to a client with active tuberculosis. When should the nurse plan to obtain a tuberculin skin test? A) Immediately after the exposure. B) Within one week of the exposure. C) Four to six weeks after the exposure. D) Three months after the exposure

C) Four to six weeks after the exposure. A tuberculin skin test is effective 4 to 6 weeks after an exposure (C), so the individual with a known exposure should wait 4 to 6 weeks before having a tuberculin skin test.

The nurse knows that lab values sometimes vary for the older client. Which data should the nurse expect to find when reviewing laboratory values of an 80-year-old male? A) Increased WBC, decreased RBC. B) Increased serum bilirubin, slightly increased liver enzymes. C) Increased protein in the urine, slightly increased serum glucose levels. D) Decreased serum sodium, an increased urine specific gravity

C) Increased protein in the urine, slightly increased serum glucose levels. Rationale: In older adults, the protein found in urine slightly rises probably as a result of kidney changes or subclinical urinary tract infections. The serum glucose increases slightly due to changes in the kidney. The specific gravity declines by age 80 from 1.032 to 1.024.

The nurse is working with a 71-year-old obese client with bilateral osteoarthritis (OA) of the hips. What recommendation should the nurse make that is most beneficial in protecting the client's joints? A) Increase the amount of calcium intake in the diet. B) Apply alternating heat and cold therapies. C) Initiate a weight-reduction diet to achieve a healthy body weight. D) Use a walker for ambulation to lessen weight-bearing on the hips.

C) Initiate a weight-reduction diet to achieve a healthy body weight. Achieving a healthy weight (C) is critical to protect the joints of clients with OA. Increasing the amount of calcium in the client's diet (A) will not protect hip joints from the effects of OA. Thermal therapies may lessen pain and stiffness from OA but are not protective of the joints (B). Assistive devices such as a walker may be beneficial to help avoid falls and assist in ambulation but are not protective against OA's effects (D).

During an interview with a client planning elective surgery, the client asks the nurse, "What is the advantage of having a preferred provider organization insurance plan?" Which response is best for the nurse to provide? A) Long-term relationships with healthcare providers are more likely. B) There are fewer healthcare providers to choose from than in an HMO plan. C) Insurance coverage of employees is less expensive to employers. D) An individual can become a member of a PPO without belonging to a group.

C) Insurance coverage of employees is less expensive to employers. Rationale: The financial advantage of (C) is the feature of a PPO that is most relevant to the average consumer. The nurse must have knowledge about PPOs, which provide discounted rates to large employers who provide insurance coverage for their employees. In return, the insurance company receives a large pool of clients for their facilities. (A, B, and D) are not accurate representations of the PPO.

The nurse is planning care to prevent complication for a client with multiple myeloma. Which intervention is most important for the nurse to include? A) Safety precautions during activity. B) Assess for changes in size of lymph nodes. C) Maintain a fluid intake of 3 to 4 L per day. D) Administer narcotic analgesic around the clock.

C) Maintain a fluid intake of 3 to 4 L per day. Multiple myeloma is a malignancy of plasma cells that infiltrate bone causing demineralization and hypercalcemia, so maintaining a urinary output of 1.5 to 2 L per day requires an intake of 3 to 4 L (C) to promote excretion of serum calcium. Although the client is at risk for pathologic fractures due to diffuse osteoporosis, mobilization and weight bearing (A) should be encouraged to promote bone reabsorption of circulating calcium, which can cause renal complications. (B) is a component of ongoing assessment. Chronic pain management (D) should be included in the plan of care, but prevention of complications related to hypercalcemia is most important.

During lung assessment, the nurse places a stethoscope on a client's chest and instructs him/her to say "99" each time the chest is touched with the stethoscope. What should be the correct interpretation if the nurse hears the spoken words "99" very clearly through the stethoscope? A) This is a normal auscultatory finding. B) May indicate pneumothorax. C) May indicate pneumonia. D) May indicate severe emphysema.

C) May indicate pneumonia. Rationale: This test (whispered pectoriloquy) demonstrates hyperresonance and helps determine the clarity with which spoken words are heard upon auscultation. Normally, the spoken word is not well transmitted through lung tissue, and is heard as a muffled or unclear transmission of the spoken word. Increased clarity of a spoken word is indicative of some sort of consolidation process (e.g., tumor, pneumonia) (C), and is not a normal finding (A). When lung tissue is filled with more air than normal, the voice sounds are absent or very diminished (e.g., pneumothorax, severe emphysema) (B and D).

An 85-year-old male client comes to the clinic for his annual physical exam and renewal of antihypertensive medication prescriptions. The client's radial pulse rate is 104 beats/minute. Which additional assessment should the nurse complete? A) Palpate the pedal pulse volume. B) Count the brachial pulse rate. C) Measure the blood pressure. D) Assess for a carotid bruit.

C) Measure the blood pressure. Elderly clients who take antihypertensive medications often experience side effects, such as hypotension, which causes tachycardia, a compensatory mechanism to maintain adequate cardiac output, so the client's blood pressure (C) should be determined. (A, B, and D) are less likely to provide data related to the client's tachycardia.

The nurse is assessing a client who smokes cigarettes and has been diagnosed with emphysema. Which finding should the nurse expect this client to exhibit? A) A decreased total lung capacity. B) Normal arterial blood gases. C) Normal skin coloring. D) An absence of sputum.

C) Normal skin coloring. The differentiation between the "pink puffer" and the "blue bloater" is a well-known method of differentiating clients exhibiting symptoms of emphysema (normal color but puffing respirations) from those exhibiting symptoms of chronic bronchitis (edematous, cyanotic, shallow respirations) (C). Total lung capacity is increased in emphysema since these clients have hyperinflated lungs (A). Arterial blood gases are typically abnormal (B). (D) is indicative of bronchitis, while clients with emphysema usually have copious amounts of thick white sputum.

An elderly male client comes to the geriatric screening clinic complaining of pain in his left calf. The nurse notices a reddened area on the calf of his right leg which is warm to the touch and suspects it might be thrombophlebitis. Which type of pain should further confirm this suspicion? A) Pain in the calf awakening him from a sound sleep. B) Calf pain on exertion which stops when standing in one place. C) Pain in the calf upon exertion which is relieved by rest and elevating the extremity. D) Pain upon arising in the morning which is relieved after some stretching and exercise.

C) Pain in the calf upon exertion which is relieved by rest and elevating the extremity. Thrombophlebitis pain is relieved by rest and elevation of the extremity (C). It typically occurs with exercise at the site of the thrombus, and is aggravated by placing the extremity in a dependent position, such as standing in one place (B). (A and D) describe pain that is not common with thrombophlebitis.

A nurse is preparing a teaching plan for a client who is post-menopausal. Which measure is most important for the nurse to include to prevent osteoporosis? A) Take a multivitamin daily. B) Use only low fat milk products. C) Perform weight resistance exercises. D) Bicycle for at least 3 miles every day.

C) Perform weight resistance exercises. Weight bearing on the skeletal system stimulates bone formation, so recommending weight resistance exercises (C) is most important in the prevention of osteoporosis in post-menopausal women. Although (A, B, and D) provide common health maintenance behaviors, weight bearing exercise provides the best preventive measure in preventing calcium mobilization out of the bone

A 26-year-old male client with Hodgkin's disease is scheduled to undergo radiation therapy. The client expresses concern about the effect of radiation on his ability to have children. What information should the nurse provide? A) The radiation therapy causes the inability to have an erection. B) Radiation therapy with chemotherapy causes temporary infertility. C) Permanent sterility occurs in male clients who receive radiation. D) The client should restrict sexual activity during radiotherapy

C) Permanent sterility occurs in male clients who receive radiation. Low sperm count and loss of motility are seen in males with Hodgkin's disease before any therapy. Radiotherapy often results in permanent aspermia, or sterility (C). (A, B, and D) are inaccurate.

A client who was in a motor vehicle collision was admitted to the hospital and the right knee was placed in skeletal traction. The nurse has documented this nursing diagnosis in the client's medical record: "Potential for impairment of skin integrity related to immobility from traction." Which nursing intervention is indicated based on this diagnosis statement? A) Release the traction q4h to provide skin care. B) Turn the client for back care while suspending traction. C) Provide back and skin care while maintaining the traction. D) Give back care after the client is released from traction.

C) Provide back and skin care while maintaining the traction. (C) indicates that back care is performed while traction is left intact, which is the correct intervention for maintaining skin integrity. Maintaining skin integrity and providing back care is difficult when a client is in traction, but it cannot be delayed until the client is removed from traction (D). The nurse should never release the traction (A and B)

A client has been taking oral corticosteroids for the past five days because of seasonal allergies. Which assessment finding is of most concern to the nurse? A) White blood count of 10,000 mm3. B) Serum glucose of 115 mg/dl. C) Purulent sputum. D) Excessive hunger.

C) Purulent sputum. Rationale: Steroids cause immunosuppression, and a purulent sputum (C) is an indication of infection, so this symptom is of greatest concern. Oral steroids may increase (A) and often cause (D). (B) may remain normal, borderline, or increase while taking oral steroids.

A neurologist prescribes a magnetic resonance imaging (MRI) of the head STAT for a patient. Which data warrants immediate intervention by the nurse concerning this diagnostic test? A) Elevated blood pressure. B) Allergy to shell fish. C) Right hip replacement. D) History of atrial fibrillation.

C) Right hip replacement. The magnetic field generated by the MRI is so strong that metal-containing items are strongly attracted to the magnet. Because the hip joint is made of metal, a lead shield must be used during the procedure. Elevated blood pressure, an allergy to shell fish, and a history of atrial fibrillation would not affect the MRI.

A client with multiple sclerosis has experienced an exacerbation of symptoms, including paresthesias, diplopia, and nystagmus. Which instruction should the nurse provide? A) Stay out of direct sunlight. B) Restrict intake of high protein foods. C) Schedule extra rest periods. D) Go to the emergency room immediately

C) Schedule extra rest periods. Exacerbations of the symptoms of MS occur most commonly as the result of fatigue and stress. Extra rest periods should be scheduled (C) to reduce the symptoms. (A, B, and D) are not necessary.

Which finding should the nurse report to the healthcare provider for a client with a circumferential extremity burn? A) Full thickness burns rather than partial thickness. B) Supinates extremity but unable to fully pronate the extremity. C) Slow capillary refill in the digits with absent distal pulse points. D) Inability to distinguish sharp versus dull sensations in the extremity

C) Slow capillary refill in the digits with absent distal pulse points A circumferential burn can form an eschar that results from burn exudate fluid that dries and acts as a tourniquet as fluid shifts occur in the interstitial tissue. As edema increases tissue pressure, blood flow to the distal extremity is compromised, which is manifested by slow capillary refill and absent distal pulses (C), so the healthcare provider should be notified about any compromised circulation that requires escharotomy. Although eschar formation occurs more readily over full thickness burns (A), the circumferential location of the burn is most likely to constrict underlying structures. Limited movement (B) is often due to pain. (D) may be related to the depth of the burn.

When preparing a client who has had a total laryngectomy for discharge, which instruction is most important for the nurse to include in the discharge teaching? A) Recommend that the client carry suction equipment at all times. B) Instruct the client to have writing materials with him at all times. C) Tell the client to carry a medic alert card stating that he is a total neck breather. D) Tell the client not to travel alone.

C) Tell the client to carry a medic alert card stating that he is a total neck breather. It is imperative that total neck breathers carry a medic alert notice (C) so that if they have a cardiac arrest, mouth-to-neck breathing can be done. Mouth-to-mouth resuscitation will not help them. They do not need to carry (A) nor refrain from (D). There are many alternative means of communication for clients who have had a laryngectomy; depending on (B) is probably the least effective. How do you know he can read and write?

A client with early breast cancer receives the results of a breast biopsy and asks the nurse to explain the meaning of staging and the type of receptors found on the cancer cells. Which explanation should the nurse provide? A) Lymph node involvement is not significant. B) Small tumors are aggressive and indicate poor prognosis. C) The tumor's estrogen receptor guides treatment options. D) Stage I indicates metastasis.

C) The tumor's estrogen receptor guides treatment options. Treatment decisions (C) and prediction of prognosis are related to the tumor's receptor status, such as estrogen and progesterone receptor status which commonly are welldifferentiated, have a lower chance of recurrence, and are receptive to hormonal therapy. Tumor staging designates tumor size and spread of breast cancer cells into axillary lymph nodes, which is one of the most important prognostic factors in early-stage breast cancer, not (A). Larger tumors are more likely to indicate poor prognosis, not (B). Stage I indicates the cancer is localized and has not spread systemically (D).

What discharge instruction is most important for a client after a kidney transplant? A) Weigh weekly. B) Report symptoms of secondary Candidiasis. C) Use daily reminders to take immunosuppressants. D) Stop cigarette smoking.

C) Use daily reminders to take immunosuppressants. After renal transplantation, acute rejection is a risk for several months, so immunosuppressive therapy, such as corticosteroids and azathioprine (Imuran), is essential in preventing rejection, so the priority instruction includes measures, such as daily reminders (C), to ensure the client takes these medications regularly. Daily weights, not weekly (A), provides a better indicator of weight gain related to rejection. Although fungal infections related to the immunosuppression should be reported (B), it is more important to ensure medication compliance to prevent rejection. Although smoking (D) increases the risk of atherosclerotic vascular disease which is common in clients with an organ transplant, (C) remains the priority

The nurse is planning care for a client with newly diagnosed diabetes mellitus that requires insulin. Which assessment should the nurse identify before beginning the teaching session? A) Present knowledge related to the skill of injection. B) Intelligence and developmental level of the client. C) Willingness of the client to learn the injection sites. D) Financial resources available for the equipment

C) Willingness of the client to learn the injection sites. If a client is incapable or does not want to learn, it is unlikely that learning will occur, so motivation is the first factor the nurse should assess before teaching (C). To determine learning needs, the nurse should assess (A), but this is not the most important factor for the nurse to assess. (B and D) are factors to consider, but not as vital as (C).

The nurse is planning care for a client with newly diagnosed diabetes mellitus that requires insulin. Which assessment should the nurse identify before beginning the teaching session? A) Present knowledge related to the skill of injection. B) Intelligence and developmental level of the client. C) Willingness of the client to learn the injection sites. D) Financial resources available for the equipment.

C) Willingness of the client to learn the injection sites. Rationale: If a client is incapable or does not want to learn, it is unlikely that learning will occur, so motivation is the first factor the nurse should assess before teaching (C). To determine learning needs, the nurse should assess (A), but this is not the most important factor for the nurse to assess. (B and D) are factors to consider, but not as vital as (C).

Which information about mammograms is most important to provide a post-menopausal female client? A) Breast self-examinations are not needed if annual mammograms are obtained. B) Radiation exposure is minimized by shielding the abdomen with a lead-lined apron. C) Yearly mammograms should be done regardless of previous normal x-rays. D) Women at high risk should have annual routine and ultrasound mammograms.

C) Yearly mammograms should be done regardless of previous normal x-rays. The current breast screening recommendation is a yearly mammogram after age 40 (C). Breast self-exam (A) continues to be a priority recommendation for all women because a small lump (or tumor) is often first felt by a woman before a mammogram is obtained. The radiation exposure from a mammogram is low, so (B) is not normally provided. The frequency of using routine and ultrasound mammograms (D) in women with high-risk variables, such as a history of breast cancer, the presence of BRC1 and BRC2 genes, or 2 first-degree relatives with breast cancer, should be recommended and followed closely by the healthcare provider.

The nurse should be correct in withholding a dose of digoxin in a client with congestive heart failure without specific instruction from the healthcare provider if the client's A) serum digoxin level is 1.5. B) blood pressure is 104/68. C) serum potassium level is 3. D) apical pulse is 68/min.

C) serum potassium level is 3. Hypokalemia (C) can precipitate digitalis toxicity in persons receiving digoxin which will increase the chance of dangerous dysrhythmias (normal potassium level is 3.5 to 5.5 mEq/L). The therapeutic range for digoxin is 0.8 to 2 ng/ml (toxic levels= >2 ng/ml); (A) is within this range. (B) would not warrant the nurse withholding the digoxin. The nurse should withhold the digoxin if the apical pulse is less than 60/min (D).

A client has taken steroids for 12 years to help manage chronic obstructive pulmonary disease (COPD). When making a home visit, which nursing function is of greatest importance to this client? Assess the client's A) pulse rate, both apically and radially. B) blood pressure, both standing and sitting. C) temperature. D) skin color and turgor.

C) temperature. Rationale: It is very important to check the client's temperature (C). Infection is the most common factor precipitating respiratory distress. Clients with COPD who are on maintenance doses of corticosteroids are particularly predisposed to infection. (A and B) are important data for baseline and ongoing assessment, but they are not as important as temperature measurement for this client who is taking steroids. Assessment of skin color and turgor is less important (D).

The nurse is providing discharge instructions to a client who has undergone a left orchiectomy for testicular cancer. Which statement indicates that the client understands his post-operative care and prognosis? A) I should continue to perform testicular self examination (TSE) monthly on my remaining testicle. B) I should wear an athletic supporter and cup to prevent testicular cancer in my remaining testicle. C) I should always use a condom because I am at increased risk for acquiring a sexually transmitted disease. D) I should make sure my sons know how to perform TSE because they are at increased risk for this type of

Correct Answer(s): A Although testicular cancer protocols, such as surgery, radiation, or chemotherapy, focus on the primary site of testicular cancer, these treatments do not reduce the risk of testicular cancer in the remaining testicle, so early recognition is the best prevention. The client's understanding is reflected in the statement to perform monthly TSE for changes in size, shape, or consistency of the testis that may indicate early cancer (A). Although an athletic support (B) protects the testicle from trauma, it does not address the client's understanding of self-care. The client's risk of STD is not related to a history of testicular cancer, but to direct exposure (C). Although the client's sons should learn TSE (D), the client should continue to TSE himself.

When observing a client for symptoms of a large bowel obstruction, the nurse should assess for which finding? A) Distention of the lower abdomen. B) Nausea with profuse vomiting. C) Upper abdominal discomfort. D) Fluid and electrolyte imbalances.

Correct Answer(s): A Among findings characteristic of a large bowel obstruction is the distention of the lower abdomen (A). (B, C, and D) are findings associated with small bowel obstruction.

The nurse is caring for a client with syndrome of inappropriate antidiuretic hormone (SIADH). This condition is most often related to which predisposing condition? A) Small cell lung cancer. B) Active tuberculosis infection. C) Hodgkin's lymphoma. D) Tricyclic antidepressant therapy.

Correct Answer(s): A Cancer is the most common cause of the syndrome of inappropriate antidiuretic hormone (SIADH), with small cell lung cancer (A) being the most common cancer that increases ADH, which causes dilutional hyponatremia and fluid retention. (B, C, and D) are also possible causes, but secondary to CNS trauma or disease.

The home health nurse is assessing a client with terminal lung cancer who is receiving hospice care. Which activity should be assigned to the hospice practical nurse (PN)? A) Administer medications for pain relief, shortness of breath, and nausea. B) Clarify family members' feelings about the meaning of client behaviors and symptoms. C) Develop a plan of care after assessing the needs of the client and family. D) Teach the family to recognize restlessness and grimacing as signs of client discomfort.

Correct Answer(s): A Hospice care provides symptom management and pain control during the dying process and enhances the quality of life for a client who is terminally ill. Administering medication and monitoring for therapeutic and adverse effects (A) is within the scope of practice for the PN. Nursing actions that require the skills of the RN include assessing and clarifying the feelings of family members (B), planning care (C), and teaching symptom recognition (D).

A client who is admitted to the coronary care unit with a myocardial infarction (MI) begins to develop increased pulmonary congestion, an increase in heart rate from 80 to 102 beats per minute, and cold, clammy skin. What action should the nurse implement? A) Notify the healthcare provider. B) Increase the IV flow rate. C) Place the client in the supine position. D) Prepare the client for an emergency echocardiography

Correct Answer(s): A Increased pulmonary congestion, increased heart rate, and cold, clammy skin in a client with a myocardial infarction indicate impending cardiogenic shock related to heart failure, a common complication of MI. The healthcare provider should be notified immediately (A) for emergency interventions of this life-threatening complication. Increasing the IV rate (B) increases the cardiac workload and contributes to cardiac decompensation. The client should be elevated to a Fowler's to semi-Fowler's position, not (C). Although an emergency echocardiography (D) should be performed, the healthcare provider should be notified for differentiating diagnosis.

A client with osteoarthritis requests information from the nurse about what type of exercise regimen would be most beneficial for him. The nurse should communicate which information? A) Low impact exercise, walking, swimming and water aerobics. B) Repetitive strength-building exercises with weights or resistance bands. C) Circuit training alternating with frequent rest periods. D) High-impact aerobic exercise

Correct Answer(s): A Low impact exercises such as walking or swimming (A), that do not cause further harm to damaged joints, are most beneficial to clients with osteoarthritis. Strengthbuilding exercises, circuit training, and high-impact aerobics (B, C and D) may cause too much stress on the joint areas and subsequently increase inflammation and damage

Which signs and symptoms are associated with arterial insufficiency? A) Pallor, intermittent claudication. B) Pedal edema, brown pigmentation. C) Blanched skin, lower extremity ulcers. D) Peripheral neuropathy, cold extremities.

Correct Answer(s): A Pallor and intermittent claudication (A) are signs related to stage II of peripheral vascular disease, which results in arterial insufficiency. (B) are signs related to venous insufficiency. (C) are not specific to arterial disease. Although (D) may be related to complications of diabetes mellitus resulting in poor circulation, arterial insufficiency causes impaired perfusion resulting in hypoxic pain or intermittent claudication.

The severity of diabetic retinopathy is directly related to which condition? A) Poor blood glucose control. B) Neurological effects of diabetes. C) Susceptibility to infection. D) Uncontrolled hypertension.

Correct Answer(s): A Poor glucose control (A) worsens diabetic retinopathy, where as tight glucose control can lessen its severity. (B, C, and D) do not affect the severity of diabetic retinopathy.

A male client with sickle cell anemia, who has been hospitalized for another health problem, tells the nurse he has had an erection for over 4 hours. What action should the nurse implement first? A) Notify the client's healthcare provider. B) Document the finding in the client record. C) Prepare a warm enema solution for rectal instillation. D) Obtain a large bore needle for aspiration of the corpora cavernosa.

Correct Answer(s): A Priapism, a urologic emergency, is common during sickle cell crisis due to sickle cells clogging the microcirculation in the penis, causing a reduction of blood flow and oxygenation to the penis, so the healthcare provider should be notified immediately (A). Documentation (B) is not the first action that should be taken. Treatment may consist of noninvasive measures such as applying ice to the penis, instilling a warm solution enema to increase outflow in the corpora cavernosa (C) and giving pain medications, but (A) has priority. If noninvasive measures do not work, (D) is implemented by the healthcare provider

Which condition is associated with an over-secretion of renin? A) Hypertension. B) Diabetes mellitus. C) Diabetes insipidus. D) Alzheimer's disease.

Correct Answer(s): A Renin is an enzyme synthesized and secreted by the juxtaglomerular cells of the kidney in response to renal artery blood volume and pressure changes. Low renal perfusion stimulates the release of renin, which is converted by angiotensinogen into angiotensin I, which causes the secretion of aldosterone, resulting in renal reabsorption of sodium, water, and subsequently increases blood pressure (A). (B, C, and D) are not directly related to renin over-secretion.

When teaching a client with breast cancer about the prescribed radiation therapy for treatment, what information is important to include? A) Dry, itchy skin changes may occur. B) There is a possibility of long bone pain. C) Permanent pigment changes to the breast may result. D) A low-residue diet may be ordered to reduce the likelihood of diarrhea

Correct Answer(s): A Side effects from radiation to the breast most often include temporary skin changes such as: dryness, tenderness, redness, swelling, and pruritis (A). (B, C, and D) are not found in this situation.

Which action should the nurse implement on the scheduled day of surgery for a client with type 1 diabetes mellitus (DM)? A) Obtain a prescription for an adjusted dose of insulin. B) Administer an oral anti-diabetic agent. C) Give an insulin dose using parameters of a sliding scale. D) Withhold insulin while the client is NPO.

Correct Answer(s): A Stressors, such as surgery, increase serum glucose levels. A client with type 1 DM who is NPO for scheduled surgery should receive a prescribed adjusted dose of insulin (A). (B, C, and D) are not indicated.

The nurse is caring for a client scheduled to undergo insertion of a percutaneous endoscopic gastrostomy (PEG) tube. The client asks the nurse to explain how a PEG tube differs from a gastrostomy tube (GT). Which explanation best describes how they are different? A) Method of insertion. B) Location of the tubes. C) Diameter of the tubes. D) Procedure for feedings.

Correct Answer(s): A The best explanation of how a PEG tube differs from a GT is by the method of insertion (A). GT insertion involves making an incision in the wall of the abdomen and suturing the tube to the gastric wall. A PEG tube is inserted with endoscopic visualization through the esophagus into the stomach and then pulled through a stab wound in the abdominal wall. (B, C, and D) identify commonalities.

Which intervention should the nurse implement that best confirms placement of an endotracheal tube (ET) tube? A) Use an end-tidal CO2 detector. B) Auscultate for bilateral breath sounds. C) Obtain pulse oximeter reading. D) Check symmetrical chest movement

Correct Answer(s): A The end-tital carbon dioxide detector indicates the presence of CO2 by a color change or a number (A), which is evidence that the ET is in the trachea, not the esophagus. Other assessments, such breath sounds (B), pulse oximetry (C) and chest movement (D), are methods to evaluate the effectiveness of ventilation and oxygenation, but do not measure CO2 in expired air from the ET.

During the initial outbreak of genital herpes simplex for a female client, what should be the nurse's primary focus in planning care? A) Promotion of comfort. B) Prevention of pregnancy. C) Instruction in condom use. D) Information about transmission.

Correct Answer(s): A The initial outbreak of genital herpes simplex in a woman causes severe discomfort. Promotion of comfort is the first priority (A). Prevention of pregnancy (B), instruction in condom use (C), and information about transmission (D) are all important to teach the client, but the client's ability to learn is hindered until the pain is controlled.

A client with a fractured right radius reports severe, diffuse pain that has not responded to the prescribed analgesics. The pain is greater with passive movement of the limb than with active movement by the client. The nurse recognizes that the client is most likely exhibiting symptoms of which condition? A) Acute compartment syndrome. B) Fat embolism syndrome. C) Venous thromboembolism. D) Aseptic ischemic necrosis.

Correct Answer(s): A These signs are specific indications of Acute Compartment Syndrome (A), and should be treated as an emergency situation. The signs do not indicate (B, C, or D).

The nurse is assessing a postmenopausal woman who is complaining of urinary urgency and frequency and stress incontinence. She also reports difficulty in emptying her bladder. These complaints are most likely due to which condition? A) Cystocele. B) Bladder infection. C) Pyelonephritis. D) Irritable bladder

Correct Answer(s): A This constellation of signs in a postmenopausal woman are characteristic of a cystocele (A). These symptoms are not characteristic of (B, C, or D).

What information should the nurse include in a teaching plan about the onset of menopause? (Select all that apply). A) Smoking. B) Oophorectomy with hysterectomy. C) Early menarche. D) Cardiac disease. E) Genetic influence. F) Chemotherapy exposure.

Correct Answer(s): A, B, C, E, F Correct responses are (A, B, C, E, and F). Menopausal symptoms are related to the cessation of ovarian function. Factors influencing the onset of menopause include smoking (A), genetic influences (E), early menarche (C), surgical removal (B), and exposure to chemotherapy agents and radiation (F). Cardiovascular disease (D) is unrelated.

The nurse is preparing a client for orthopedic surgery on the left leg and completing a safety checklist before transport to the operating room. Which items should the nurse remove from the client? (Select all that apply.) A) Nail polish. B) Hearing aid. C) Wedding band. D) Left leg brace. E) Contact lenses. F) Partial dentures.

Correct Answer(s): A, B, E, F (Correct selections are A, B, E, and F). The removal of nail polish (A) provides a more accurate pulse oximetry readings and evaluation of capillary refill. Hearing aids (B), contact lenses (E), and partial dentures (F) are removed to prevent damage, loss or misplacement, or injury during surgery. (C and D) should remain with the client.

The nurse is assessing a client admitted from the emergency room with gastrointestinal bleeding related to peptic ulcer disease (PUD). Which physiological factors can produce ulceration? (Select all that apply.) A) Vagal stimulation. B) An increased level of stress. C) Decreased duodenal inhibition. D) Hypersecretion of hydrochloric acid. E) An increased number of parietal cells.

Correct Answer(s): A, C, D, E Correct selections are (A, C, D, and E). Hypersecretion of gastric juices (D) and an increased number of parietal cells (E) that stimulate secretion are most often the causes of ulceration. Vagal stimulation (A) and decreased duodenal inhibition (C) also increase the secretion of caustic fluids. An increased stress level is not physiologic and is not a direct cause of ulceration (B).

A client with a recent history of blood in his stools is scheduled for a proctosigmoidoscopy. The nurse should implement which protocols to prepare the client for this procedure? (Select all that apply.) A) Obtain consent for the procedure. B) Initiate preoperative sedation. C) Begin fast the morning of the procedure. D) Administer an enema before the procedure. E) Provide a clear-liquid diet 48 hours before the procedure

Correct Answer(s): A, C, D, E Correct selections are (A, C, D, and E). The usual preoperative preparation for proctosigmoidoscopy entails obtaining the client's consent to the procedure (A), a clear-liquid diet for 24 to 48 hours prior to the procedure (E), administration of an enema (D), and fasting (C) on the morning of the procedure. Preoperative sedation is not the norm for this procedure (B), although some healthcare providers administer a mild tranquilizer.

Which findings are within expected parameters of a normal urinalysis for an older adult? (Select all that apply.) A) pH 6. B) Nitrate small. C) Protein small. D) Sugar negative. E) Bilirubin negative. F) Specific gravity 1.015.

Correct Answer(s): A, D, E, F Correct selections are (A, D, E, and F). (A) is within the normal pH range for urine. Glucosuria and bilirubinuria are abnormal and should be negative upon urinalysis (D and E). Normal changes associated with aging include decreased creatinine clearance and decreased concentrating and diluting abilities which influence the normal range of urine specific gravity, 1.001 to 1.035. Although common health problems associated with aging include renal insufficiency, urinary incontinence, urinary tract infection (B and C), and enlarged prostate, these are indicative of pathology which should be treated.

A client with a chronic infection of Hepatitis C virus (HCV) is scheduled for a liver biopsy. Which intervention should the nurse perform after the procedure? A) Progress activity as soon as possible. B) Assess for signs of bleeding and hypovolemia. C) Place the client in the left lateral position. D) Monitor blood pressure, pulse and breathing every 4 hours.

Correct Answer(s): B Assessment for signs of bleeding (B) should be implemented because internal bleeding is the greatest risk following a liver biopsy. Having the client placed a right lateral position, not left (C) applies pressure at the site. Because of the increased risk for bleeding, a gradual return to normal activities over 1-2 days is desired (A). Monitoring vital signs at 1-2 hour intervals (D) for 6-8 hours after the procedure is recommended to detect pneumothorax, hemothorax, or other internal bleeding.

The nurse directs an unlicensed assistive personnel (IAP) to obtain the vital signs for a client who returns to the unit after having a mastectomy for cancer. What information should the nurse provide the UAP? A) Elevate the arm with an IV infusing on the operative side with a pillow. B) Apply the blood pressure cuff to the arm on the unoperative side. C) Position the arm on the operative side close to the body. D) Collect a fingerstick blood specimen from the arm on the operative side.

Correct Answer(s): B Blood pressure readings should be obtained from the arm on the un-operative side (B) to reduce the risk of injury of the extremity that may have compromised lymphatic drainage postoperatively. The arm on the operative side of the mastectomy should be elevated on a pillow above the level of the right atrium to facilitate lymphatic drainage, not (C). An IV infusion (A) or blood specimen collection (D) should not involve the use of the arm on the operative side.

An older female client is admitted with atrophic vaginitis and perineal cutaneous candidiasis. What is the priority nursing diagnosis for this client? A) Risk for injury. B) Impaired comfort. C) Disturbed body image. D) Ineffective health maintenance.

Correct Answer(s): B In menopausal women, the vaginal mucous membrane responds to low estrogen levels causing the vaginal walls to become thinner, drier, and susceptible to infection which leads to atrophic vaginitis. Perineal cutaneous candidiasis contributes to other manifestations of vaginal infections, such as vaginal irritation, burning, pruritus, increased leukorrhea, bleeding, and dyspareunia, and support the primary nursing diagnosis, Impaired comfort (B). Risk for injury (A), body image (C), and ineffective health maintenance (D) are secondary and linked to impaired comfort.

The nurse is providing instructions about log rolling to a client who returns to the postoperative unit after a lumbar laminectomy. Which explanation should the nurse give the client about this technique? A) Helps to minimize pain and anxiety. B) Maintains correct spinal alignment to protect the surgical area. C) Prevents dizziness while stabilizing the spine. D) Allows the nurse to move the client freely without assistance.

Correct Answer(s): B Log-rolling technique maintains the spine in a straight superior-inferior plane that aligns the spine without movement while protecting the surgical area (B), which is especially important when the procedure involves bone grafts that may several weeks for the bone to fuse. (A) is best managed with client teaching, preparation before procedures, relaxation techniques, and pain management, not log-rolling. (C and D) are not accurate

The nurse is caring for a client with human immunodeficiency virus (HIV) infection who develops Mycobacterium avium complex (MAC). What is the most significant desired outcome for this client? A) Free from injury of drug side effects. B) Return to pre-illness weight. C) Adequate oxygenation. D) Maintenance of intact perineal skin.

Correct Answer(s): B MAC is an opportunistic infection that presents as a tuberculosis-like pulmonary process. MAC is a major contributing factor to the development of wasting syndrome, so the most significant desired outcome is the client's return to a pre-illness weight (B) using oral, enteral, or parenteral supplementation as needed. Drug schedules and side effects (A) remain a life long management problem. Client outcomes for adequate oxygenation (C) are often dependent on management of anemia, maintenance of activities without fatigue, and supplemental oxygen to prevent hypoxia. Skin integrity (D) is dependent upon resolution of diarrhea, which is not as significant as optimal nutrition.

A client with sickle cell anemia is admitted with severe abdominal pain and the diagnosis is sickle cell crisis. What is the most important nursing action to implement? A) Limit the client's intake of oral fluids and food. B) Evaluate the effectiveness of narcotic analgesics. C) Encourage the client to ambulate as tolerated. D) Teach the client about prevention of crises.

Correct Answer(s): B Pain management is the priority for a client during sickle cell crisis. Continuous narcotic analgesics are the mainstay of pain control, which should be evaluated (B) frequently to determine if the client's pain is adequately controlled. (A, C, and D) are not indicated at this time.

A client's family asks why their mother with heart failure needs a pulmonary artery (PA) catheter now that she is in the intensive care unit (ICU). What information should the nurse include in the explanation to the family? A) A central monitoring system reduces the risk of complications undetected by observation. B) A pulmonary artery catheter measures central pressures for monitoring fluid replacement. C) Pulmonary artery catheters allow for early detection of lung problems. D) The healthcare provider should explain the many reasons for it's use.

Correct Answer(s): B Pulmonary artery catheters are used to measure central pressures and fluid balance (B). Even though all clients in the ICU require close monitoring, they do not all need a PA catheter (A). PA lines do not detect pulmonary problems (C). (D) avoids the family's question.

A client who is receiving a whole blood transfusion develops chills, fever, and a headache 30 minutes after the transfusion is started. The nurse should recognize these symptoms as characteristic of what reaction? A) A mild allergic reaction. B) A febrile transfusion reaction. C) An anaphylactic transfusion reaction. D) An acute hemolytic transfusion reaction.

Correct Answer(s): B Symptoms of a febrile reaction (B) include sudden chills, fever, headache, flushing and muscle pain. An allergic reaction (A) is the response of histamine release which is characterized by flushing, itching, and urticaria. An anaphylactic reaction (C) exhibits an exaggerated allergic response that progresses to shock and possible cardiac arrest. An acute hemolytic reaction (D) presents with fever and chills, but is hallmarked by the onset of low back pain, tachycardia, tachypnea, vascular collapse, hemoglobinuria, dark urine, acute renal failure, shock, cardiac arrest, and even death.

A Korean-American client, who speaks very little English, is being discharged following surgery. Which nurse should the nurse manager assign to provide the discharge instructions for the client? A) A graduate registered nurse (RN) with three weeks of experience. B) The registered nurse (RN) case manager for the unit with 1 years' experience. C) A floating registered nurse (RN) with five years of nursing experience. D) A Korean-American practical nurse (PN) with six years of nursing experience.

Correct Answer(s): B The RN case manager (B) is the best qualified nurse to assess and provide discharge educational needs, obtain resources for the client, enhance coordination of care, and prevent fragmentation of care. The RN graduate (A) lacks the experience to provide individualized and complete discharge instructions. The float nurse (C) lacks case management expertise to advocate adequately for the client, coordinate care, and provide community resources. It is not in the scope of practice for the PN (D) to give discharge instructions.

The nurse is caring for a client who is two days postoperative. Which observation should alert the nurse to call the Rapid Response Team (RRT)? A) Fresh bleeding noted on abdominal surgical wound dressing. B) Pulse change from 85 to160 beats/minute lasting more than 10 minutes. C) Temperature of 103.1° F and white blood cell (WBC) count of 16,000 mm3. D) Weakness, diaphoresis, complaints of feeling faint. BP 100/56 mm Hg

Correct Answer(s): B The RRT should be called to intervene for a client with an acute life-threatening change, such as (B). (A) indicates possible hemorrhage and needs further investigation and monitoring. (B) indicates an infection and (D) may indicate post-operative diuresis with corresponding hypotension. Although these symptoms need prompt collaborative attention, they can be dealt with through normal channels such providing supportive care and calling the healthcare provider.

The nurse assesses a long-term resident of a nursing home and finds the client has a fungal infection (candidiasis) beneath both breasts. To prevent nosocomial infection, which protocol should the nurse review with the rest of the staff? A) Follow contact isolation procedures. B) Wash hands after caring for the client. C) Wear gloves when providing personal care. D) Restrict pregnant staff or visitors into the room.

Correct Answer(s): B The organism Candida albicans, that causes this infection, is part of the normal flora on the skin of most adults. Good handwashing (B) is all that is needed to prevent nosocomial spread. (A) is not necessary. Standard precautions (C) should be used during moisture management and when applying a prescription for the active infection, but the client is not considered a risk to others (D).

A man who smokes two packs of cigarettes a day wants to know if smoking is contributing to the difficulty that he and his wife are having getting pregnant. What information is best for the nurse to provide? (Select all that apply.) A) Only marijuana cigarettes affect sperm count. B) Smoking can decrease the quantity and quality of sperm. C) The first semen analysis should be repeated to confirm sperm counts. D) Cessation of smoking improves general health and fertility. E) Sperm specimens should be collected in 2 subsequent days.

Correct Answer(s): B, C, D Correct selections are (B, C, and D). Use of tobacco, alcohol, and marijuana may affect sperm counts (B). Sperm counts vary from day to day and are dependent on emotional and physical status and sexual activity, so a single analysis may be inconclusive (C). A minimum of two analyses should be performed several weeks apart to assess male fertility, not (E). (A and D) contain inaccurate information.

When caring for a client with a percutaneous endoscopic gastrostomy (PEG) tube, what protocols should the nurse implement for intermittent feedings? (Select all that apply.) A) Assessing residual amounts once a day. B) Keeping the head of the bed elevated 30 degrees. C) Changing the enteral-feeding bag every 24 hours. D) Checking the placement of the tube by means of gastric aspiration. E) Flushing the tube with 50 ml of normal saline solution after each feeding.

Correct Answer(s): B, C, D, E (B, C, D, and E) are correct. Keeping the head of the bed elevated 30 degrees (B), changing the enteral-feeding bag every 24 hours (C), checking the placement of the tube by means of gastric aspiration (D), and flushing the tube with 50 ml of normal saline solution after each feeding (E) are interventions used to provide care of the client with a PEG tube. Residual amounts should be assessed prior to each feeding, not once daily (A).

What is the priority nursing action while caring for a client on a ventilator when an electrical fire occurs in the intensive care unit? A) Tell another staff member to bring extinguishing equipment to the bedside. B) Close the doors to the client's area when attempting to extinguish the fire. C) Use a bag-valve-mask resuscitator while removing the client from the area. D) Implement an emergency protocol to remove the client from the ventilator.

Correct Answer(s): C A client on a ventilator should have respirations maintained with a manual bag-valve-mask resuscitator while being moved away from the oxygen wall outlet and fire source (C). (A, B, and D) are not the priority in maintaining client safety during a fire in the client care area.

The nurse is teaching a client about precautions for a new prescription for lovastatin (Mevacor). Which symptom should the nurse instruct the client to report to the healthcare provider immediately? A) Terrible nightmares. B) Increased nocturia. C) Severe muscle pain. D) Visual disturbances.

Correct Answer(s): C A potential, serious side effect of statin therapy that is used to lower both LDL-C and triglyceride levels is rhabdomyolysis, which is manifested by severe muscle pain and aching (C). (A) is a side effect, but not life threatening. (B) is not related to statin therapy. Blurred vision (D) is a transient side effect that does not need immediate medical treatment

A client who returns to the unit after having a percutaneous transluminal coronary angioplasty (PTCA) complains of acute chest pain. What action should the nurse implement next? A) Inform the healthcare provider. B) Obtain a 12-lead electrocardiogram. C) Give a sublingual nitroglycerin tablet. D) Administer prescribed analgesic.

Correct Answer(s): C After a percutaneous transluminal coronary angioplasty (PTCA), a client who experiences acute chest pain may be experiencing cardiac ischemia related to restenosis, stent thrombosis, or acute coronary syndrome involving any coronary artery. The first action is to administer nitroglycerin (C) to dilate the coronary arteries and increase myocardial oxygenation. Then, (A, B, and D) are implemented.

A 24-year-old female client diagnosed with a human papillomavirus infection (HPV) is angry at her exboyfriend and says she is not going to tell him that he is infected. What response is best for the nurse to provide? A) You do not have to tell him because this is not a reportable disease. B) Because there is no cure for this disease, telling him is of no benefit to him or to you. C) Even though you are angry, he should be told, so he can take precautions to prevent the spread of infection. D) You should tell him, so he can feel as guilty and miserable as you do now, knowing that you have this disease.

Correct Answer(s): C Anger is a common emotional reaction when confronted with the diagnosis of a STI, and often lay blame and project this anger at the sexual partner. Although HPV is not a reportable disease in many states, all contacts should be informed of the infection, treatment, transmission, and precautions to minimize infecting others (C). (A and B) provide false information and increase the risk of complications and transmission. (D) is not therapeutic.

A client with a history of hypertension, myocardial infarction, and heart failure is admitted to the surgical intensive care unit after coronary artery bypass surgery graft (CABG). The nurse determines the client's serum potassium level is 4.5 mEq/L. What action should the nurse implement? A) Notify the healthcare provider. B) Decrease the IV solution flow rate. C) Document the finding as the only action. D) Administer potassium replacement as prescribed.

Correct Answer(s): C Coronary artery bypass surgery graft (CABG) places a client at risk for hypokalemia from hemodilution, nasogastric suction, or diuretic therapy, so the serum potassium level is maintained between 4 and 5 mEq/L to avoid dysrhythmias. Documentation of the normal finding (C) is indicated at this time. Continued monitoring of the client should anticipate the onset of complications that may require (A, B, and C).

A 48-year-old client with endometrial cancer is being discharged after a total hysterectomy and bilateral salpingo-oophorectomy. Which client statement indicates that further teaching is needed? A) Well, I don't have to worry about getting pregnant anymore. B) I can't wait to go on the cruise that I have planned for this summer. C) I know I will miss having sexual intercourse with my husband. D) I have asked my daughter to stay with me next week after I am discharged

Correct Answer(s): C Further teaching is needed in response to the client's misunderstanding of sexuality after a hysterectomy that is reflected in statement (C). The client's knowledge about reproduction (A), a positive outlook with plans for the future (B), and her anticipated need for assistance and support during recovery (D) indicate she understands the present status of her recovery.

A client who is admitted to the emergency department with a possible tension pneumothorax after a motor vehicle collision is having multiple diagnostic tests. Which finding requires immediate action by the nurse? A) Serum amylase of 132 units/L. B) Serum sodium of 134 mEq/L. C) Chest x-ray indicating a mediastinal shift. D) Abdominal x-ray air throughout intestines.

Correct Answer(s): C Immediate action is required for findings of a mediastinal shift (C), which can precipitate life threatening cardiovascular collapse as the great cardiac vessels become kinked and compressed due to increasing air entrapment in the pleural space. (A and B) do not warrant action (normal levels for serum sodium level 135 to 145 mEq/L and amylase 30 to 220 units/L). (D) describes a normal gas pattern in the intestines.

A client in the preoperative holding area receives a prescription for midazolam (Versed) IV. The nurse determines that the surgical consent form needs to be signed by the client. Which action should the nurse implement? A) Give the drug and allow the client to read and sign the consent form. B) Counter-sign the client's initials on the consent form after giving the drug. C) Withhold the drug until the client validates understanding of the surgical procedure and signs the consent form. D) Call the healthcare provider to explain the surgical procedure before the client signs the consent.

Correct Answer(s): C Midazolam, a benzodiazepine sedative, is commonly used for conscious-sedation intraoperatively and interferes with the client's cognition and level of consciousness, so the consent form should be signed before the drug is administered (C). The validity of legal documents will be in question if a client signs them while under the influence of any central nervous system depressant drug (A and B). If indicated, (D) may need to be implemented but should be determined before the client arrives to the preoperative area.

A client with rheumatoid arthritis is prescribed piroxicam (Feldene), a nonsteroidal anti-inflammatory drug (NSAID). Which effect is characteristic of (NSAIDs) used for treating rheumatoid arthritis? A) Production of replacement cartilage is stimulated. B) Further destruction of the articular cartilage is prevented. C) Inflammation is reduced by inhibiting prostaglandin synthesis. D) Bradykinin is inhibited, thereby reducing acute and chronic pain.

Correct Answer(s): C Nonsteroidal anti-inflammatory drugs (NSAIDs), used for treating rheumatoid arthritis, inhibit the synthesis of prostaglandins and relieve associated pain (C), but they do not generate new cartilage (A). NSAIDs are not an effective treatment to inhibit bradykinin (D). Joint destruction is not preventable with this disease process (B).

A client with type II diabetes arrives at the clinic with a blood glucose of 50 mg/dL. The nurse provides the client with 6 ounces of orange juice. In 15 minutes, the client's capillary glucose is 74 mg/dL. What action should the nurse take? A) Obtain a specimen for serum glucose level. B) Administer insulin per sliding scale. C) Provide cheese and bread to eat. D) Collect a glycosylated hemoglobin specimen.

Correct Answer(s): C Once blood glucose is greater than 70 mg/dl, the client should eat a regularly scheduled meal or a snack to prevent hypoglycemia from recurring (C). Blood glucose has just been checked and a serum level is not indicated at this time (A). The blood glucose does not indicate a need for insulin (B) which may further exacerbate a hypoglycemic response. A glycosylated hemoglobin (hemoglobin A1C) level is not indicated at this time (D)

The nurse is caring for a client who returns to the unit following a colonoscopy. Which finding should the nurse report to the healthcare provider immediately? A) Large amounts of expelled flatus with mucus. B) Tympanic abdomen and hyperactive bowel sounds. C) Increased abdominal pain with rebound tenderness. D) Complaint of feeling weak with watery diarrheal stools.

Correct Answer(s): C Positive rebound tenderness (C) may be an indication of peritonitis or perforation and needs follow-up immediately. Clients typically experience a large amount of flatus (A) and may have mucus from bowel irritation from the procedure. A tympanic abdomen on percussion and hyperactive bowel sounds are typical post procedure findings (B). Weakness and watery stools are a result from the preparation and are common symptoms experienced after a colonoscopy (D).

A client who had abdominal surgery two days ago has prescriptions for intravenous morphine sulfate 4 mg every 2 hours and a clear liquid diet. The client complains of feeling distended and has sharp, cramping gas pains. What nursing intervention should be implemented? A) Obtain a prescription for a laxative. B) Withhold all oral fluid and food. C) Assist the client to ambulate in the hall. D) Administer the prescribed morphine sulfate.

Correct Answer(s): C Postoperative abdominal distention is caused by decreased peristalsis as a result of handling the intestine during surgery, limited dietary intake before and after surgery, and anesthetic and analgesic agents. Peristalsis is stimulated and distention minimized by implementing early and frequent ambulation (C). Based on the client's status, laxatives (A) or withholding dietary progression (B) are not indicated at this time. Although pain management should be implemented (D), another analgesic prescription may be needed because morphine reduces intestinal motility and contributes to the client's gas pains.

The nurse is caring for a client who had an excision of a malignant pituitary tumor. Which findings should the nurse document that indicate the client is developing syndrome of inappropriate antidiuretic hormone (SIADH)? A) Hypernatremia and periorbial edema. B) Muscle spasticity and hypertension. C) Weight gain with low serum sodium. D) Increased urinary output and thirst.

Correct Answer(s): C SIADH most frequently occurs when cancer cells manufacture and release ADH, which is manifested by water retention causing weight gain and hyponatremia (C). Other manifestations include oliguria, weakness, not (A, B, and D), anorexia, nausea, vomiting, personality changes, seizures, decrease in reflexes, and coma.

The nurse is caring for a male client who had an inguinal herniorrhaphy 3 hours ago. The nurse determines the client's lower abdomen is distended and assesses dullness to percussion. What is the priority nursing action? A) Assessment of the client's vital signs. B) Document the finding as the only action. C) Determine the time the client last voided. D) Insert a rectal tube for the passage of flatus.

Correct Answer(s): C Swelling at the surgical site in the immediate postoperative period can impact the bladder and prostate area causing the client to experience difficulty voiding due to pressure on the urethra. To provide additional data supporting bladder distention, the last time the client voided (C) should be determined next. Documentation (B) should be made, but the client's distended bladder requires additional intervention. (A and D) are not priority actions based on the client's abdominal findings.

The nurse is caring for a client receiving tamoxifen (Nolvadex) for the treatment of breast cancer. Which action should the nurse include in the client's plan of care? A) Increase fluid intake. B) Monitor sodium chloride intake. C) Assist the client in coping with hot flashes. D) Encourage milk products to increase calcium intake.

Correct Answer(s): C Tamoxifen, an estrogen receptor blocking agent, can cause hot flashes, so (C) should be included in the plan of care. Increasing fluid intake (A), monitoring sodium intake (B), and encouraging milk products to increase calcium intake (D) are not related to the care of a client receiving tamoxifen.

A client is admitted to the Emergency Department with a tension pneumothorax. Which assessment should the nurse expect to identify? A) An absence of lung sounds on the affected side. B) An inability to auscultate tracheal breath sounds. C) A deviation of the trachea toward the side opposite the pneumothorax. D) A shift of the point of maximal impulse to the left, with bounding pulses

Correct Answer(s): C Tension pneumothorax is caused by rapid accumulation of air in the pleural space, causing severely high intrapleural pressure. This results in collapse of the lung, and the mediastinum shifts toward the unaffected side, which is subsequently compressed (C). (A, B, and D) are not demonstrated with a tension pneumothorax.

The nurse is caring for a client who is admitted with a hemorrhagic stroke. Which nursing action should be included in the plan of care? A) Perform active range of motion three times daily. B) Monitor for Battle's sign every four hours. C) Teach measures to avoid the Valsalva maneuver. D) Maintain the head of bed in a flat position.

Correct Answer(s): C The Valsalva maneuver, straining with bowel movements while holding one's breath, increases intracerebral pressure (ICP) which may induce bleeding or rupture of cerebral blood vessels (C). Passive, not active ROM (A) is performed to avoid ICP, bleeding, and rupture. Battle's sign (B), bruising noted behind the ear, is a manifestation that may be seen with a basilar skull fracture, not hemorrhagic stroke. The flat position for the head of bed is avoided (D) because it increases venous congestion and ICP

A client who has a chronic cough with blood-tinged sputum returns to the unit after a bronchoscopy. What nursing interventions should be implemented in the immediate post-procedural period? A) Keep the client on bed rest for eight hours. B) Check vital signs every 15 minutes for two hours. C) Allow the client nothing by mouth until the gag reflex returns. D) Encourage fluid intake to promote elimination of the contrast media.

Correct Answer(s): C The nasal pharynx and oral pharynx are anesthetized with local anesthetic spray prior to bronchoscopy, and the bronchoscope is coated with lidocaine (Xylocaine) gel to inhibit the gag reflex and prevent laryngeal spasm during insertion. The client should be NPO until the client's gag reflex returns (C) to prevent aspiration from any oral intake or secretions. (A, B, and D) are not indicated after bronchoscopy.

Which finding should the nurse identify as an indication of carbon monoxide poisoning in a client who experienced a burn injury during a house fire? A) Pulse oximetry reading of 80%. B) Expiratory stridor and nasal flaring. C) Cherry red color to the mucous membranes. D) Presence of carbonaceous particles in sputum.

Correct Answer(s): C The saturation of hemoglobin molecules with carbon monoxide and the subsequent vasodilation induce a cherry red color of the mucous membranes (C) in a client who experienced a burn injury during a house fire. Super heated air or smoke inhalation damage the lining of the airways which causes swelling, decreased oxygenation (A), and an expiratory stridor (B). Mouth breathing during the fire allows the inhalation of soot that is seen as particles in the client's sputum (D).

The nurse is caring for a client with multiple trauma after a motor vehicle collision. The nurse learns that the client has secondary syphilis. What precaution should the nurse implement? A) A mask should be worn by anyone entering the client's room. B) Handwashing is required before and after contact with the client. C) Gloves should be worn during direct contact with the client's skin. D) No precautions in addition to standard precautions are necessary.

Correct Answer(s): C The secondary stage of syphilis is a systemic bloodborne disease that presents with skin lesions and rashes that may drain the highly contagious spirochete, so gloves should be worn during direct contact with the client's skin (C). Although contact with the client's blood or lesions should be avoided, masks (A) are not indicated. (B and D) are implemented for all clients.

A client with acute osteomyelitis has undergone surgical debridement of the diseased bone and asks the nurse how long will antibiotics have to be administered. Which information should the nurse communicate? A) Oral antibiotics for 2 to 4 months, then for dental procedure prophylaxis. B) Parenteral antibiotics for 4 to 6 weeks, then oral antibiotics for up to 1 year. C) Parenteral antibiotics for 4 to 8 weeks, then oral antibiotics for 4 to 8 weeks. D) Parenteral antibiotics for 2 to 3 weeks, then oral antibiotics for 4 weeks.

Correct Answer(s): C Treatment of acute osteomyelitis requires administration of high doses of parenteral antibiotics for 4 to 8 weeks, followed by oral antibiotics for another 4 to 8 weeks (C). (A, B, and D) are incorrect antibiotic schedules for osteomyelitis.

What is the primary nursing diagnosis for a client with asymptomatic primary syphilis? A) Acute pain. B) Risk for injury. C) Sexual dysfunction. D) Deficient knowledge.

Correct Answer(s): D An asymptomatic client with primary syphilis is most likely unaware of this disease, so to prevent transmission to others and recurrence in the client, the priority nursing diagnosis is deficient knowledge (D). Asymptomatic primary syphilis is not painful, so (A) is not applicable at this time. Although the client is at risk for injury (B) and sexual dysfunction (C) related to complications, teaching the client about transmission and treatment is instrumental in preventing the progression to systemic secondary or tertiary syphilis.

The nurse is assessing a middle-aged male client for risk factors related to chronic illness. Which finding should the nurse assess further? A) Thinning hair and dry scalp. B) Increase in appetite and taste-bud acuity. C) Increase in muscle tone but decreased muscle strength. D) Increase in abdominal fat deposits

Correct Answer(s): D An increase in the abdominal girth (D) may be indicative of the onset of metabolic syndrome, which places the client at risk for cardiac disease and requires further assessment. During middle adulthood, common findings include thinning hair, dry skin and scalp (A), changes in taste bud acuity (B), and muscle size and strength (C), which are consistent with normal system functioning during aging

Which sexually transmitted infection (STI) should the nurse include in a client's teaching plan about the risk for cervical cancer? A) Neisseria gonorrhea. B) Chlamydia trachomatis. C) Herpes simplex virus. D) Human papillomavirus.

Correct Answer(s): D Human papillomavirus (D) is known to alter cervical epithelium cytology, which is consistent with early changes of cervical cancer. Although STIs (A, B, and C) place the client at risk for exposure to HPV, these are likely to place the client at risk for pelvic inflammatory disease, infertility sequela, and painful reoccurrence.

A client is admitted after blunt abdominal injury. Which assessment finding requires immediate action by the nurse? A) Radiating abdominal pain with left lower quadrant palpation. B) Grimacing after palpation of the right hypochondriac region. C) Rebound tenderness with abdominal palpation. D) Bluish periumbilical skin discoloration.

Correct Answer(s): D Immediate action is indicated for intraperitoneal hemorrhage which causes periumbilical discoloration (D) and indicates the presence of a splenic rupture, a life threatening complication of blunt abdominal injury. (A, B, and C) indicate inflammation of the appendix or gallbladder but do not represent an acute finding as a result of blunt abdominal trauma.

The PET (positron emission tomography) scan is commonly used with oncology clients to provide for which diagnostic information? A) A description of inflammation, infection, and tumors. B) Continuous visualization of intracranial neoplasms. C) Imaging of tumors without exposure to radiation. D) An image that describes metastatic sites of cancer

Correct Answer(s): D PET scans provide information regarding certain diseases of the heart (determination of tissue viability), brain (dementia, Parkinson's disease), and early detection of tumors and their aggressiveness. This diagnostic test scans the body to detect the spread of cancer (metastasis) (D). (A, B, and C) are not the purpose of PET.

A middle-aged male client asks the nurse what findings from his digital rectal examination (DRE) prompted the healthcare provider to prescribe a repeat serum prostatic surface antigen (PSA) level. What information should the nurse provide? A) A uniformly enlarged prostate is benign prostatic hypertrophy that occurs with aging. B) The spongy or elastic texture of the prostate is normal and requires no further testing. C) An infection is usually present when the prostate indents when a finger is pressed on it. D) Stony, irregular nodules palpated on the prostate should be further evaluated.

Correct Answer(s): D PSA levels are prescribed to screen for prostatic cancer which is often detected by DRE and manifested as small, hard, or stony, irregularly-shaped nodules on the surface of the prostate (D). Although PSA levels are prescribed for routine screening, the findings suggestive of BPH (A), normal texture (B) or infection (C) do not suggest cancer of the prostate, which requires further evaluation.

Several hours after surgical repair of an abdominal aortic aneurysm (AAA), the client develops left flank pain. The nurse determines the client's urinary output is 20 ml/hr for the past 2 hours. The nurse should conclude that these findings support which complication? A) Infection. B) Hypovolemia. C) Intestinal ischemia. D) Renal artery embolization.

Correct Answer(s): D Postoperative complications of surgical repair of AAA are related to the location of resection, graft, or stent placement along the abdominal aorta. Embolization of a fragment of thrombus or plaque from the aorta into a renal artery (D) can compromise blood flow in one of the renal arteries, resulting in renal ischemia that precipitates unilateral flank pain. Intraoperative blood loss or rupture of the aorta anastomosis can cause acute renal failure related to hypovolemia (B), which involves both kidneys and causing bilateral flank pain. (A and C) are not associated with these symptoms.

A client with osteoarthritis receives a prescription for Naproxen (Naprosyn). Which potential side effect should the nurse provide to the client about this medication? A) Sensitivity to sunlight. B) Muscle fasciculations. C) Increased urinary frequency. D) Gastrointestinal disturbance.

Correct Answer(s): D Prostaglandin synthesis inhibitors such as naproxen can have gastrointestinal side effects such as nausea and gastric burning (D). It is recommended that this drug be taken with food to avoid gastrointestinal upset. Naproxen (Naprosyn) does not cause sensitivity to sunlight (A), muscle fasciculations (B), or urinary frequency (C).

The nurse is planning preoperative teaching for a client who will undergo a radical neck dissection and total laryngectomy. Which information has the greatest priority for this client? A) Prognosis after treatment is excellent. B) Techniques for esophageal speech are relatively easy to learn with practice. C) The stoma should never be covered after this type of surgery. D) There is a radical change in appearance as a result of this surgery.

Correct Answer(s): D Radical neck dissection is the removal of lymphatic drainage channels and nodes, sternocleidomastoid muscle, spinal accessory nerve, jugular vein, and submandibular area. The overall outcome of this type of surgery causes the neck to be disfigured, so (D) should be a priority in the care of this client. (A, B, and C) are included, but the client's concern for (D) is the priority.

A client is admitted for complaints of chest pain and aching for the past 4 days. The results for serum creatine kinase-MB (CK-MB) and troponin levels are obtained. What rationale should the nurse use to evaluate the laboratory findings? A) Serum myoglobin levels are needed to confirm myocardial damage. B) The most reliable indicator of myocardial necrosis is serum CK-MB. C) Serum cardiac markers are inconclusive in determining myocardial injury after waiting several days. D) Myocardial damage that occurred several days earlier is best validated by serum troponin levels.

Correct Answer(s): D Serum CK-MB and troponin are the two most important serum cardiac markers for confirming myocardial infarction. CK-MB begins to rise in the first 3 to 12 hours after the myocardial infarction, peaks in 24 hours, and returns to normal in 2 to 3 days. The troponin level rises as quickly but remains elevated for 2 weeks (D). (A, B, and C) do not provide the most significant data.

A client who is receiving the sixth unit of packed red blood cell transfusion is demonstrating signs and symptoms of a febrile, nonhemolytic reaction. What assessment finding is most important for the nurse to identify? A) Increased anxiety since the transfusion began. B) Drowsiness after receiving diphenhydramine (Benadryl). C) Complaints of feeling cold. D) Flushed skin and headache.

Correct Answer(s): D The most common type of reaction is a febrile, nonhemolytic blood transfusion reaction related to leukocyte incompatibility, which causes chills, fever, headache, and flushing (D). Some clients are anxious (A) about the risk of blood-borne infections, but the client's response to the release of inflammatory and immunologic mediators can potentially lead to bronchospasm and circulatory collapse. Drowsiness (B) is an expected symptom after diphenhydramine administration. (C) is often a sensory response to environmental temperatures or the administration of cold blood

A female client with hyperesthesia on the oncology unit is using a transcutaneous electrical nerve stimulation (TENS) unit for chronic pain. Which nursing activity should the nurse implement instead of delegating to a practical nurse (PN)? A) Determine the client's level of discomfort using a pain rating scale. B) Ask the client about her past experience with chronic pain. C) Observe the client's facial expressions for pain and discomfort. D) Evaluate the client's ability to adjust the voltage to control pain.

Correct Answer(s): D The oncology nurse has the knowledge and experience with the use of a transcutaneous electrical nerve stimulation (TENS) unit for chronic pain relief, so the nurse should evaluate the client's skill in effectively controlling the pain by adjusting the voltage (D). The PN can collect data about the client's pain (A, B, and C).

The nurse is giving discharge instructions to a client with chronic prostatitis. What instruction should the nurse provide the client to reduce the risk of spreading the infection to other areas of the client's urinary tract? A) Wear a condom when having sexual intercourse. B) Avoid consuming alcohol and caffeinated beverages. C) Empty the bladder completely with each voiding. D) Have intercourse or masturbate at least twice a week.

Correct Answer(s): D The prostate is not easily penetrated by antibiotics and can serve as a reservoir for microorganisms, which can infect other areas of the genitourinary tract. Draining the prostate regularly through intercourse or masturbation (D) decreases the number of microorganisms present and reduces the risk for further infection from stored contaminated fluids. (A, B, and C) do not reduce the risk of spreading the infection internally.

The healthcare provider prescribes high-protein, highfat, low-carbohydrate diet with limited fluids during meals for a client recovering from gastric surgery. The client asks the nurse what the purpose is for this type of diet. Which rationale should be included in the nurse's explanation to this client? A) It is quickly digested. B) It does not cause diarrhea. C) It does not dilate the stomach. D) It is slow to leave the stomach.

Correct Answer(s): D This type of diet is slowly digested and is slow to leave the stomach (D). Because of its density from proteins and fats, and the reduction of fluids with the meal, the possibility of dumping syndrome is reduced. (A, B, and C) are incorrect rationales.

A client with Ménière's disease is incapacitated by vertigo and is lying in bed grasping the side rails and staring at the television. Which nursing intervention should the nurse implement? A) Encourage fluids to 3000 ml per day. B) Change the client's position every two hours. C) Keep the head of the bed elevated 30 degrees. D) Turn off the television and darken the room.

Correct Answer(s): D To decrease the client's vertigo during an acute attack of Ménière's disease, any visual stimuli or rotational movement, such as sudden head movements or position changes, should be minimized. Turning off the television and darkening the room (D) minimize fluorescent lights, flickering television lights, and distracting sound. (A, B, and C) are ineffective in managing the client's symptoms.

The nurse is assessing a client with a cuffed tracheostomy tube in place who is breathing spontaneously. To evaluate if the client can tolerate cuff deflation to promote speaking and swallowing, what action should the nurse implement? A) Ask the client to try to speak. B) Assess for respiratory distress. C) Auscultate for pulmonary crackles after the client drinks a small amount of clear water. D) Observe the client for coughing colored sputum after drinking a small amount of colored water.

Correct Answer(s): D To evaluate the risk for aspiration after the cuff is deflated, the client should be instructed to swallow a small amount of colored water, then observed for coughing up colored sputum (D), or the tracheostomy should be suctioned for the presence of colored water. (A) does not determine if the client is at risk to aspirate oral intake. Large volumes of oral intake are more likely to cause respiratory distress (B) or crackles (C), and should not be used to evaluate the client's risk for aspiration.

A male client with a prostatic stent is preparing for discharge. What should the nurse ensure the client understands? A) Ongoing antibiotic therapy is needed for one year. B) The client should not undergo magnetic resonance imaging. C) Increased frequency of assessment for prostatic cancer is needed. D) The client should not be catheterized through the stent for at least three months.

Correct Answer(s): D To prevent complications, the client should be cautioned against catheterization through the stent for three months after stent placement (D). Long term antibiotic use for one year (A) is not a part of illness management. There is no contraindication for magnetic resonance imaging (B). Frequent assessment of prostate health is part of client teaching for health promotion (C), but is not increased because of the stent placement.

The nurse is caring for a client with a small bowel obstruction. The client is vomiting foul smelling fecallike material. What action should the nurse implement? A) Administer antiemetics every 2 to 3 hours. B) Position on the left side with knees drawn up. C) Encourage ice chips sparingly. D) Give IV fluids with electrolytes.

Correct Answer(s): D When the bowel is obstructed, electrolytes and fluids are not absorbed, so parenteral fluids with sodium chloride, bicarbonate, and potassium should be administered (D). (A and C) are contraindicated. (B) may or may not be a position of comfort for the client. The nurse should implement (D).

Duplex scanning confirms the presence of a deep venous thrombosis for a client with swelling and pain of the lower leg. While the client is receiving continuous heparin infusion, what actions should the nurse implement? A) Avoid any intramuscular medications to prevent localized bleeding. B) Have vitamin K available in the event the client begins to bleed. C) Notify the healthcare provider if the partial thromboplastin time is greater than 50 seconds. D) Start instruction for self-administered SC heparin injections for long-term home therapy.

Correct Answer: A

A client with a markedly distended bladder is diagnosed with hydronephrosis and left hydroureter after an IV pyelogram. The nurse catheterizes the client and obtains a residual urine volume of 1650 ml. This finding supports which pathophysiological cause of the client's urinary tract obstruction? A) Obstruction at the urinary bladder neck. B) Ureteral calculi obstruction. C) Ureteropelvic junction stricture. D) Partial post-renal obstruction due to ureteral stricture.

Correct Answer: A Hydroureter (dilation of the renal pelvis), vesicoureteral reflux (backward movement of urine from the lower to upper urinary tracts), and hydronephrosis (dilation or enlargement of the renal pelvis and calyces) result from post-renal obstruction which can consequently result in chronic pyelonephritis and renal atrophy. Ascending urinary reflux occurs when normal ureteral peristaltic pressure is met with an increase in urinary pressure occurring during bladder filling if the urinary bladder neck is obstructed (A). A large residual urine does not occur with (B, C, and D) because the urine cannot get to the bladder.

Which client is at highest risk for chronic kidney disease (CKD) secondary to diabetes mellitus (DM)? A) Type 1 DM and a serum hemoglobin-A1c of 3.5%. B) Type 1 DM and retinopathy and mild vision loss. C) Type 2 DM and hypertension controlled by metoprolol. D) Type 2 DM and a history of morbid obesity for 5 years.

Correct Answer: B Diabetic retinopathy and nephropathy are related to prolonged hyperglycemia and hypertension which damage the microvasculature of the eyes and kidneys, so a client with Type 1 DM and retinopathy is most likely to develop nephropathy (B) and CKD. (A) is demonstrating compliance with therapy (H-A1c target level is below 7%), which indicates tight glucose control and reduces the risk for microvascular complications. The client with controlled hypertension (C) is less likely to develop CKD, although metoprolol, a beta-adrenergic receptor antagonist, can mask the signs of hypoglycemia. A client with Type 2 DM is more likely at risk for complications associated with chronic obesity (D).

While assessing the hair and scalp of an adult client, the nurse notes that the client has dry, brittle hair. Which information should the nurse obtain first? A) Unexplained weight gain. B) Current hair care practices. C) Family history of alopecia. D) Absence of axillary hair

Correct Answer: B Dry and brittle hair may be a result of hair treatments such as hair dyes, rinses, permanents, straighteners, or frequent blow-drying (B). Although an unexplained weight gain (A) could be related to hypothyroidism, which causes hair to become dry and brittle, assessing current hair care practices should be determined first because of the prevalent use of cosmetic products. Next, a family history of alopecia (C) and absence of axillary hair (D) should be assessed to identify other problems contributing to hair abnormalities, such as nutritional deficiencies, endocrine dysfunction, or genetic predisposition.

After receiving chemotherapy 2 weeks ago, a male client with acute leukemia is admitted for blood transfusions because his hemoglobin is 6 gm/dl. After toileting, the client returns to bed and his oxygen saturation is measured at 82%. The nurse increases the O2 per nasal cannula from 3 to 4 liters per minute. What intervention should the nurse implement next? A) Collect blood for hemoglobin and hematocrit. B) Start the first transfusion of blood. C) Insert an indwelling urinary catheter. D) Encourage alternate rest periods with activity.

Correct Answer: B The hemoglobin of 6 gm/dl (normal is 14 to 18 gm/dl in males) and the 82% O2 saturation (normal is 96 to 100%) indicates the client is hypoxic, so the first transfusion of blood should be started (B). (A) should be obtained after the client is transfused to evaluate its effectiveness. (C) is not indicated at this time. (D) should be included in the plan of care, but is not as essential as (B) at this time.

A 24-year-old female client diagnosed with a human papillomavirus infection (HPV) is angry at her exboyfriend and says she is not going to tell him that he is infected. What response is best for the nurse to provide? A) You do not have to tell him because this is not a reportable disease. B) Because there is no cure for this disease, telling him is of no benefit to him or to you. C) Even though you are angry, he should be told, so he can take precautions to prevent the spread of infection. D) You should tell him, so he can feel as guilty and miserable as you do now, knowing that you have this disease.

Correct Answer: C Anger is a common emotional reaction when confronted with the diagnosis of a STI, and often lay blame and project this anger at the sexual partner. Although HPV is not a reportable disease in many states, all contacts should be informed of the infection, treatment, transmission, and precautions to minimize infecting others (C). (A and B) provide false information and increase the risk of complications and transmission. (D) is not therapeutic.

While the nurse obtains a male client's history, review of systems, and physical examination, the client tells the nurse that his breast drains fluid secretions from the nipple. The nurse should seek further evaluation of which endocrine gland function? A) Posterior pituitary and testes. B) Adrenal medulla and adrenal cortex. C) Hypothalamus and anterior pituitary. D) Parathyroid and islets of Langerhans.

Correct Answer: C CoBreast fluid and milk production are induced by the presence of prolactin secreted from the anterior pituitary gland, which is regulated by the hypothalamus' secretion of prolactin-inhibiting hormone in both men and women. Further evaluation of the hypothalamus and the anterior pituitary gland (C) should provide additional information about the secretions or lactation. Evaluation of (A, B, or D) do not support a physiologic mechanism or pathology related to mammary discharge

The nurse is caring for a client with a nursing diagnosis of, "Infection, risk for, related to inadequate primary defenses as evidenced by surgical incision and IV access." What nursing intervention should the nurse implement? A) Limit visitors to immediate family to decrease exposure to infection. B) Maintain "clean" technique in the change of wound dressing and IV site. C) Assess and document skin condition around the incision and IV site at each shift. D) Require the use of a face mask by staff when providing care requiring close contact.

Correct Answer: C Early identification of infection leads to prompt treatment and decreased nosocomial transmission to others, so the condition of any invasive lines or breaks in the skin should be assessed and documented (C) during each shift. (A and D) are not indicated for care of this client. Sterile technique is used in the dressing change or IV site change, not (B).

While auscultating the lungs of a client who is being mechanically ventilated, the nurse hears coarse, snoring sounds over the upper anterior chest with clear sounds over the other lung fields. Based on these assessment findings, which action should the nurse take? A) Notify respiratory therapy immediately for a PRN bronchodilator treatment. B) Obtain a prescription to increase the tidal volume setting on the ventilator. C) Stop mechanical ventilation and re-assess the client's lung sounds bilaterally. D) Suction the client's endotracheal tube and auscultate following suctioning.

Correct Answer: D Coarse, snoring sounds (rhonchi) heard over large upper airways are frequently produced by secretions partially blocking air passages and usually disappear after suctioning (D). (A) is indicated for a bronchospasm, which typically produces wheezing or musical adventitious lung sounds. Increasing the tidal volume (B) does not help resolve the problem. Mechanical ventilators produce noise that makes lung auscultation difficult, but removal of the ventilator to listen to breath sounds (C) is contraindicated, as this may reduce oxygenation.

The nurse is providing postoperative instructions for a female client after a mastectomy. Which information should the nurse include in the teaching plan? (Select all that apply.) A) Empty surgical drains once a week using procedure gloves. B) Report inflammation of the incision site or the affected arm. C) Wear clothing with snug sleeves over the arm on the operative side. D) Avoid lifting more than 4.5 kg (10 lb) or reaching above her head.

Correct answers include (B and D). Part of the client's teaching plan should include reporting evidence of inflammation at the incision or of the affected arm (B), and to avoid lifting or reaching (D), which places the client at risk for injury to the extremity that may have compromised lymphatic drainage. The client should be instructed to empty surgical drains daily, not (A). Activity that decreases circulation (C) in the affected arm, such as carrying a handbag over the shoulder, wearing tight clothing, or tight jewelry, should be avoided.

What assessment findings should the nurse identify before referring a client for further evaluation to rule out skin cancer? (Select all that apply.) A) White patches. B) Cherry angiomas. C) Border irregularity. D) Lesion with asymmetry. E) Lesion with color variations. F) Lesion of 3 to 5 mm diameter.

Correct selections are (C, D, and E). ABCDE is the acronym used by the American Cancer Society (ACS) to monitor lesions needing further evaluation to rule out skin cancer: A for asymmetry of the lesion (D); B for irregular border (C); C for color, usually dark (E); D for diameter equal to or greater than 6 mm; and E for elevation. A lesion with any of the characteristics of ABCDE should be evaluated by a healthcare provider. (A) lack the color variable. (B) are raised, dome-shaped, benign clusters of blood vessels that do not require treatment. Lesions of 3 to 5 mm diameter are small and may be monitored instead of treated (F).

In planning care for a client with an acute stroke resulting in right-sided hemiplegia, which positioning should the nurse should use to maintain optimal functioning? A) Mid-Fowler's with knees supported. B) Supine with trochanter rolls to the hips. C) Sim's position alternated with right lateral position q2 hours. D) Left lateral, supine, brief periods on the right side, and prone

D After an acute stroke, a positioning and turning schedule that minimizes lying on the affected side, which can impair circulation and cause pain, and includes the prone position (D) to help prevent flexion contractures of the hips, prepares the client for optimal functioning and ambulating. (A, B, and C) do not maintain the client for optimal functioning.

A client reports unprotected sexual intercourse one week ago and is worried about HIV exposure. An initial HIV antibody screen (ELISA) is obtained. The nurse teaches the client that seroconversion to HIV positive relies on antibody production by B lymphocytes after exposure to the virus. When should the nurse recommend the client return for repeat blood testing? A) 6 to 18 months. B) 1 to 12 months. C) 1 to 18 weeks. D) 6 to 12 weeks.

D) 6 to 12 weeks. Although the HIV antigen is detectable approximately 2 weeks after exposure, seroconversion to HIV positive may take up to 6 to 12 weeks (D) after exposure, so the client should return to repeat the serum screen for the presence of HIV antibodies during that time frame. (A) will delay treatment if the client tests positive. (B and C) may provide inaccurate results because the time frame maybe too early to reevaluate the client.

A client who is HIV positive asks the nurse, "How will I know when I have AIDS?" Which response is best for the nurse to provide? A) Diagnosis of AIDS is made when you have 2 positive ELISA test results. B) Diagnosis is made when both the ELISA and the Western Blot tests are positive. C) I can tell that you are afraid of being diagnosed with AIDS. Would you like for me to call your minister? D) AIDS is diagnosed when a specific opportunistic infection is found in an otherwise healthy individual.

D) AIDS is diagnosed when a specific opportunistic infection is found in an otherwise healthy individual. AIDS is diagnosed when one of several processes defined by the CDC is present in an individual who is not otherwise immunosuppressed (D) (PCP, candidacies, Cryptococcus, cryptosporidiosis, Kaposi's sarcoma, CNS lymphomas). (A and B) identify the presence of HIV, indicating a high probability that in time the individual will develop AIDS, but do not necessarily denote the presence of AIDS. (C) is telling the client how he/she feels (afraid) and is dismissing the situation to the minister. This client is asking a question and specific medical information needs to be provided.

Which condition is considered a non-modifiable risk factor for a brain attack? A) High cholesterol levels. B) Obesity. C) History of atrial fibrillation. D) Advanced age.

D) Advanced age. Rationale: People over age 55 are a high-risk group for a brain attack because the incidence of stroke more than doubles in each successive decade of life. Non-modifiable means the client cannot do anything to change the risk factor. All the other options are modifiable risk factors.

Two days postoperative, a male client reports aching pain in his left leg. The nurse assesses redness and warmth on the lower left calf. What intervention should be most helpful to this client? A) Apply sequential compression devices (SCDs) bilaterally. B) Assess for a positive Homan's sign in each leg. C) Pad all bony prominences on the affected leg. D) Advise the client to remain in bed with the leg elevated.

D) Advise the client to remain in bed with the leg elevated. The client is exhibiting symptoms of deep vein thrombosis (DVT), a complication of immobility. The initial care includes bedrest and elevation of the extremity (D). SCDs are used to prevent thrombophlebitis, not for treatment, when a clot might be dislodged (A). Once a client has thrombophlebitis, (B) is contraindicated because of the possibility of dislodging a clot. (C) is indicated to prevent pressure ulcers, but is not a therapeutic action for thrombophlebitis.

The nurse is assisting a client out of bed for the first time after surgery. What action should the nurse do first? A) Place a chair at a right angle to the bedside. B) Encourage deep breathing prior to standing. C) Help the client to sit and dangle legs on the side of the bed. D) Allow the client to sit with the bed in a high Fowler's position.

D) Allow the client to sit with the bed in a high Fowler's position. Rationale: The first step is to raise the head of the bed to a high Fowler's position (D), which allow venous return to compensate from lying flat and vasodilating effects of perioperative drugs. (A, B, and C) are implemented after (D).

A male client with chronic atrial fibrillation and a slow ventricular response is scheduled for surgical placement of a permanent pacemaker. The client asks the nurse how this devise will help him. How should the nurse explain the action of a synchronous pacemaker? A) Ventricular irritability is prevented by the constant rate setting of pacemaker. B) Ectopic stimulus in the atria is suppressed by the device usurping depolarization. C) An impulse is fired every second to maintain a heart rate of 60 beats per minute. D) An electrical stimulus is discharged when no ventricular response is sensed

D) An electrical stimulus is discharged when no ventricular response is sensed. The artificial cardiac pacemaker is an electronic device used to pace the heart when the normal conduction pathway is damaged or diseased, such as a symptomatic dysrhythmias like atrial fibrillation with a slow ventricular response. Pacing modes that are synchronous (impulse generated on demand or as needed according to the patient's intrinsic rhythm) send an electrical signal from the pacemaker to the wall of the myocardium stimulating it to contract when no ventricular depolarization is sensed (D). (A, B, and C) do not provide accurate information

A client taking furosemide (Lasix), reports difficulty sleeping. What question is important for the nurse to ask the client? A) What dose of medication are you taking? B) Are you eating foods rich in potassium? C) Have you lost weight recently? D) At what time do you take your medication?

D) At what time do you take your medication? The nurse needs to first determine at what time of day the client takes the Lasix (D). Because of the diuretic effect of Lasix, clients should take the medication in the morning to prevent nocturia. The actual dose of medication (A) is of less importance than the time taken. (B) is not related to the insomnia. (C) is valuable information about the effect of the diuretic, but is not likely to be related to insomnia.

A client who is fully awake after a gastroscopy asks the nurse for something to drink. After confirming that liquids are allowed, which assessment action should the nurse consider a priority? A) Listen to bilateral lung and bowel sounds. B) Obtain the client's pulse and blood pressure. C) Assist the client to the bathroom to void. D) Check the client's gag and swallow reflexes.

D) Check the client's gag and swallow reflexes. Following gastroscopy, a client should remain nothing by mouth until the effects of local anesthesia have dissipated and the airway's protective reflexes, gag and swallow reflexes, have returned (D). (A, B, and C) are not the priority before reintroducing oral fluids after a gastroscopy

A female client is brought to the clinic by her daughter for a flu shot. She has lost significant weight since the last visit. She has poor personal hygiene and inadequate clothing for the weather. The client states that she lives alone and denies problems or concerns. What action should the nurse implement? A) Notify social services immediately of suspected elderly abuse. B) Discuss the need for mental health counseling with the daughter. C) Explain to the client that she needs to take better care of herself. D) Collect further data to determine whether self-neglect is occuring

D) Collect further data to determine whether self-neglect is occurring. Changes in weight and hygiene may be indicators of self=-neglect or neglect by family members. Further assessment is needed (D) before notifying social services (A) or discussing a need for counseling (B). Until further information is obtained, explanations about the client's needs are premature (C).

An elderly client is admitted with a diagnosis of bacterial pneumonia. The nurse's assessment of the client is most likely to reveal which sign/symptom? A) Leukocytosis and febrile. B) Polycythemia and crackles. C) Pharyngitis and sputum production. D) Confusion and tachycardia

D) Confusion and tachycardia. The onset of pneumonia in the elderly may be signaled by general deterioration, confusion, increased heart rate or increased respiratory rate (D). (A, B, and C) are often absent in the elderly with bacterial pneumonia.

The nurse assesses a client with advanced cirrhosis of the liver for signs of hepatic encephalopathy. Which finding should the nurse consider an indication of progressive hepatic encephalopathy? A) An increase in abdominal girth. B) Hypertension and a bounding pulse. C) Decreased bowel sounds. D) Difficulty in handwriting.

D) Difficulty in handwriting. A daily record in handwriting may provide evidence of progression or reversal of hepatic encephalopathy leading to coma (D). (A) is a sign of ascites. (B) are not seen with hepatic encephalopathy. (C) does not indicate an increase in serum ammonia level which is the primary cause of hepatic encephalopathy.

A client who has just tested positive for human immunodeficiency virus (HIV) does not appear to hear what the nurse is saying during post-test counseling. Which information should the nurse offer to facilitate the client's adjustment to HIV infection? A) Inform the client how to protect sexual and needlesharing partners. B) Teach the client about the medications that are available for treatment. C) Identify the need to test others who have had risky contact with the client. D) Discuss retesting to verify the results, which will ensure continuing contact.

D) Discuss retesting to verify the results, which will ensure continuing contact. Encouraging retesting (D) supports hope and gives the client time to cope with the diagnosis. Although post-test counseling should include education about (A, B, and C), retesting encourages the client to maintain medical follow-up and management.

What is the correct procedure for performing an ophthalmoscopic examination on a client's right retina? A) Instruct the client to look at examiner's nose and not move his/her eyes during the exam. B) Set ophthalmoscope on the plus 2 to 3 lens and hold it in front of the examiner's right eye. C) From a distance of 8 to 12 inches and slightly to the side, shine the light into the client's pupil. D) For optimum visualization, keep the ophthalmoscope at least 3 inches from the client's eye.

D) For optimum visualization, keep the ophthalmoscope at least 3 inches from the client's eye. The client should focus on a distant object in order to promote pupil dilation. The ophthalmoscope should be set on the 0 lens to begin (creates no correction at the beginning of the exam), and should be held in front of the examiner's left eye when examining the client's right eye. For optimum visualization, the ophthalmoscope should be kept within one to three inches of the client's eye (D). (A and B) describe incorrect methods for conducting an ophthalmoscopic examination. (C) should illicit a red reflex as the light travels through the crystalline lens to the retina.

Which clinical manifestation further supports an assessment of a left-sided brain attack? A) Visual field deficit on the left side. B) Spatial-perceptual deficits. C) Paresthesia of the left side. D) Global aphasia.

D) Global aphasia Rationale: Global aphasia refers to difficulty speaking, listening, and understanding, as well as difficulty reading and writing. Symptoms vary from person to person. Aphasia may occur secondary to any brain injury involving the left hemisphere. Visual field deficits, spatial-perceptual deficits, and paresthsia of the left side usually occur with right-sided brain attack.

The nurse is receiving report from surgery about a client with a penrose drain who is to be admitted to the postoperative unit. Before choosing a room for this client, which information is most important for the nurse to obtain? A) If suctioning will be needed for drainage of the wound. B) If the family would prefer a private or semi-private room. C) If the client also has a Hemovac® in place. D) If the client's wound is infected.

D) If the client's wound is infected. Rationale: Penrose drains provide a sinus tract or opening and are often used to provide drainage of an abscess. The fact that the client has a penrose drain should alert the nurse to the possibility that the client is infected. To avoid contamination of another postoperative client, it is most important to place an infected client in a private room (D). A penrose drain does not require (A). Although (B) is information that should be considered, it does not have the priority of (D). (C) is used to drain fluid from a dead space and is not important in choosing a room.

A client has a staging procedure for cancer of the breast and ask the nurse which type of breast cancer has the poorest prognosis. Which information should the nurse offer the client? A) Stage II. B) Invasive infiltrating ductal carcinoma. C) T1N0M0. D) Inflammatory with peau d'orange.

D) Inflammatory with peau d'orange. Inflammatory breast cancer, which has a thickened appearance like an orange peel (peau d'orange), is the most aggressive form of breast malignancies (D). Staging classifies cancer by the extension or spread of the disease, and (A) indicates limited local spread. (B) indicates cancer cells have spread from the ducts into the surrounding breast tissue only. TNM classification is used to indicate the extent of the disease process according to tumor size, regional spread lymph nodes involvement, and metastasis, and (C) indicates early cancer with small in situ involvement, no lymph node involvement, and no distant metastases

To decrease the risk of acid-base imbalance, what goal must the client with diabetes mellitus strive for? A) Checking blood glucose levels once daily B) Drinking 3 L of fluid per day C) Eating regularly, every 4 to 8 hours D) Maintaining blood glucose level within normal limits

D) Maintaining blood glucose level within normal limits Rationale: Maintaining blood glucose levels within normal limits is the best way to decrease the risk of acidbase imbalance. A) Blood glucose levels must be checked several times a day. B) Drinking 3 L of fluid per day is not necessary to maintain acid-base balance. C) Eating regularly is a way to achieve acid-base balance but is not the goal itself.

The nurse is preparing an adult client for an upper gastrointestinal (UGI) series. Which information should the nurse include in the teaching plan? A) The x-ray procedure may last for several hours. B) A nasogastric tube (NGT) is inserted to instill the barium. C) Enemas are given to empty the bowel after the procedure. D) Nothing by mouth is allowed for 6 to 8 hours before the study.

D) Nothing by mouth is allowed for 6 to 8 hours before the study. The client should be NPO for at least 6 hours before the UGI (D). (A) is not typical for this procedure. A NGT is not needed to instill the barium (B) unless the client is unable to swallow. A laxative, not enemas (C), is given after the procedure to help expel the barium.

During a health fair, a 72-year-old male client tells the nurse that he is experiencing shortness of breath. Auscultation reveals crackles and wheezing in both lungs. Suspecting that the client might have chronic bronchitis, which classic symptom should the nurse expect this client to have? A) Racing pulse with exertion. B) Clubbing of the fingers. C) An increased chest diameter. D) Productive cough with grayish-white sputum

D) Productive cough with grayish-white sputum. Chronic bronchitis, one of the diseases comprising the diagnosis of COPD, is characterized by a productive cough with grayish-white sputum (D), which usually occurs in the morning and is often ignored by smokers. (A) is not related to chronic bronchitis; however, it is indicative of other problems such as ventricular tachycardia and should be explored. (B and C) are symptoms of emphysema and are not consistent with the other symptoms. (C) is usually referred to as a "barrel chest."

Three weeks after discharge for an acute myocardial infarction (MI), a client returns to the cardiac center for follow-up. When the nurse asks about sleep patterns, the client tells the nurse that he sleeps fine but that his wife moved into the spare bedroom to sleep when he returned home. He states, I guess we will never have sex again after this. Which response is best for the nurse to provide? A) Sexual intercourse can be strenuous on your heart, but closeness and intimacy, such as holding and cuddling, can be maintained with your wife. B) Sexual activity can be resumed whenever you and your wife feel like it because the sexual response is more emotional rather than physical. C) You should discuss your questions about your sexual activity with your healthcare provider because sexual activity may be limited by your heart damage. D) Sexual activity is similar in cardiac workload and energy expenditure as climbing two flights of stairs and may be resumed like other activities

D) Sexual activity is similar in cardiac workload and energy expenditure as climbing two flights of stairs and may be resumed like other activities. Sexual intercourse after an MI, or acute coronary syndrome, has been found to require no more energy expenditure or cardiac stress than walking briskly up two flights of stairs (D), as long as other guidelines, such as limiting food and alcohol intake before intercourse, are followed. (A, B, and C) do not provide the best factual information to reduce the client's anxiety and misconceptions

A client asks the nurse about the purpose of beginning chemotherapy (CT) because the tumor is still very small. Which information supports the explanation that the nurse should provide? A) Side effects are less likely if therapy is started early. B) Collateral circulation increases as the tumor grows. C) Sensitivity of cancer cells to CT is based on cell cycle rate. D) The cell count of the tumor reduces by half with each dose.

D) The cell count of the tumor reduces by half with each dose. Initiating chemotherapy while the tumor is small provides a better chance of eradicating all cancer cells because 50% of cancer cells or tumor cells are killed with each dose. (A, B, and C) vary based on the type of cancer.

A 57-year-old male client is scheduled to have a stressthallium test the following morning and is NPO after midnight. At 0130, he is agitated because he cannot eat and is demanding food. Which response is best for the nurse to provide to this client? A) I'm sorry sir, you have a prescription for nothing by mouth from midnight tonight. B) I will let you have one cracker, but that is all you can have for the rest of tonight. C) What did the healthcare provider tell you about the test you are having tomorrow? D) The test you are having tomorrow requires that you have nothing by mouth tonight.

D) The test you are having tomorrow requires that you have nothing by mouth tonight. (D) is the most therapeutic statement because the nurse is responding to the client's question. (A) is not an explanation and the nurse should teach the client why eating is prohibited after midnight, rather than enforcing this requirement without an explanation for it. (B) may result in an inaccurate test result, or may cause the test to be cancelled, which could also delay diagnosis and treatment. (C) defers the responsibility for answering the client's question to the healthcare provider, when the nurse could address the situation through client teaching

Which method elicits the most accurate information during a physical assessment of an older client? A) Ask the client to recount one's health history. B) Obtain the client's information from a caregiver. C) Review the past medical record for medications. D) Use reliable assessment tools for older adults.

D) Use reliable assessment tools for older adults. Specific assessment tools (D) for an older adult, such as Older Adult Resource Services Center Instrument (OARS), minimental assessment, fall risk, depression (Geriatric Depression Scale), or skin breakdown risk (Braden Scale), consider agerelated physiologic and psychosocial changes related to aging and provide the most accurate and complete information. (A and B) are subjective and may vary in reliability based on the client's memory and caregiver's current involvement. Although (C) is a good resource to identify polypharmacy, a written record may . not be available or currently accurate

A patient admitted for a head injury develops dry skin and urine output of 600 mL/hr. Which of the following interventions should the nurse perform first? a) Assess the patient's urine specific gravity b) Slow IV fluid infusion rate c) Assess the patient's level of consciousness d) Notify the physician

a) Assess the patient's urine specific gravity Ratoinale: A urine output of 400 mL/hr after sustaining a head injury may be indicative of diabetes insipidus. The nurse should assess for low specific gravity and elevated serum osmolarity. -Diabetes insipidus is the failure to produce antidiuretic hormone due to damage to the pituitary gland from increased ICP. -Notifying the physician is appropriate after the nurse has gathered additional data.

A patient is prescribed dexamethasone (Decadron) to reduce cerebral edema after a motor vehicle accident. Which of the following assessment findings should the nurse expect if this treatment is effective? a) Increased response to stimuli b) decreased urine output c) respiration rate of 12 d) Increased blood pressure

a) Increased response to stimuli rationale: Dexamethasone (Decadron) is a corticosteroid that reduces inflammation in the brain. When effectiveness is achieved, the patient's neurological status should improve. -Decadron has little effect on blood pressure, respiration rate, and urine output.

The nurse is assessing a client with a chest tube that is attached to suction and a closed drainage system. Which finding is most important for the nurse to further assess? A) Upper chest subcutaneous emphysema. B) Tidaling (fluctuation) of fluid in the water-seal chamber. C) Constant air bubbling in the suction-control chamber. D) Pain rated 8 (0-10) at the insertion site.

A Subcutaneous emphysema (A) is a complication and indicates air is leaking beneath the skin. Tidaling in the water-seal chamber and constant bubbling with suction in the suction-control chamber (B and C) are expected findings that indicate the closed drainage system is working. Pain at the insertion site is an expected finding (D) and the prescribed analgesia should be given to assist the client to breathe deeply and facilitate lung expansion.

When planning care for a client with right renal calculi, which nursing diagnosis has the highest priority? A) Acute pain related to movement of the stone. B) Impaired urinary elimination related to obstructed flow of urine. C) Risk for infection related to urinary stasis. D) Deficient knowledge related to need for prevention of recurrence of calculi.

A) Acute pain related to movement of the stone. The nursing diagnosis of highest priority is acute pain (A), which if unresolved can represent pathology affecting renal function. Impaired urinary elimination (B), risk for infection (C), and knowledge deficit (D) are components of the plan of care with less immediacy than management of the etiology of the client's pain.

A male client who has never smoked but has had COPD for the past 5 years is now being assessed for cancer of the lung. The nurse knows that he is most likely to develop which type of lung cancer? A) Adenocarcinoma. B) Oat-cell carcinoma. C) Malignant melanoma. D) Squamous-cell carcinoma.

A) Adenocarcinoma. Adenocarcinoma is the only lung cancer not related to cigarette smoking (A). It has been found to be directly related to lung scarring and fibrosis from preexisting pulmonary disease such as TB or COPD. Both (B and D) are malignant lung cancers related to cigarette smoking. (C) is a skin cancer and is related to exposure to sunlight, not to lung problems.

A client who is receiving chemotherapy asks the nurse, "Why is so much of my hair falling out each day?" Which response by the nurse best explains the reason for alopecia? A) Chemotherapy affects the cells of the body that grow rapidly, both normal and malignant. B) Alopecia is a common side effect you will experience during long-term steroid therapy. C) Your hair will grow back completely after your course of chemotherapy is completed. D) The chemotherapy causes permanent alterations in your hair follicles that lead to hair loss

A) Chemotherapy affects the cells of the body that grow rapidly, both normal and malignant. The common adverse effects of chemotherapy (nausea, vomiting, alopecia, bone marrow depression) are due to chemotherapy's effect on the rapidly reproducing cells, both normal and malignant (A). (B and D) do not provide correct information about chemotherapy-induced alopecia. Although (D) is a true statement, it does not effectively answer the client's question.

The nurse is assessing a client who is bedfast and refuses to turn or move from a supine position. How should the nurse assess the client for possible dependent edema? A) Compress the flank and upper buttocks. B) Measure the client's abdominal girth. C) Gently palpate the lower abdomen. D) Apply light pressure over the shins.

A) Compress the flank and upper buttocks. Dependent edema collects in dependent areas, such as the flank and upper buttocks (A) of the client who is persistently flat in bed. (B) provides data about ascites (fluid collection in the abdomen), rather than dependent edema, and (C) provides data about abdominal distention. (D) provides data about the collection of dependent edema for a client whose lower extremities are often in a dependent position, such as when sitting in a chair

The nurse is interviewing a male client with hypertension. Which additional medical diagnosis in the client's history presents the greatest risk for developing a cerebral vascular accident (CVA)? A) Diabetes mellitus. B) Hypothyroidism. C) Parkinson's disease. D) Recurring pneumonia

A) Diabetes mellitus. A history of diabetes mellitus poses the greatest risk for developing a CVA (A). (B, C, and D) may place the client at some risk due to immobility, but do not present a risk as great as (A).

A client is brought to the Emergency Center after a snow-skiing accident. Which intervention is most important for the nurse to implement? A) Review the electrocardiogram tracing. B) Obtain blood for coagulation studies. C) Apply a warming blanket. D) Provide heated PO fluids.

A) Review the electrocardiogram tracing. Airway, breathing, and circulation are priorities in client assessment and treatment. Continuous cardiac monitoring is indicated (A) because hypothermic clients have an increased risk for dysrhythmias. Coagulations studies (A) and re-warming procedures (C and D) can be initiated after a review of the ECG tracing (A).

A 46-year-old female client is admitted for acute renal failure secondary to diabetes and hypertension. Which test is the best indicator of adequate glomerular filtration? A) Serum creatinine. B) Blood Urea Nitrogen (BUN). C) Sedimentation rate. D) Urine specific gravity

A) Serum creatinine. Creatinine (A) is a product of muscle metabolism that is filtered by the glomerulus, and blood levels of this substance are not affected by dietary or fluid intake. An elevated creatinine strongly indicates nephron loss, reducing filtration. (B) is also an indicator of renal activity, but it can be affected by non-renal factors such as hypovolemia and increased protein intake. (C) is a nonspecific test for acute or chronic inflammatory processes. (D) is useful in assessing hydration status, but not as useful in assessing glomerular function

The nurse formulates the nursing diagnosis of, Urinary retention related to sensorimotor deficit for a client with multiple sclerosis. Which nursing intervention should the nurse implement? A) Teach the client techniques of intermittent selfcatheterization. B) Decrease fluid intake to prevent over distention of the bladder. C) Use incontinence briefs to maintain hygiene with urinary dribbling. D) Explain that anticholinergic drugs will decrease muscle spasticity.

A) Teach the client techniques of intermittent selfcatheterization. Bladder control is a common problem for clients with multiple sclerosis. A client with urinary retention should receive instructions about self-catheterization (A) to prevent bladder distention. Adequate hydration, not (B), is important to reduce the risk of urinary tract infections by promoting elimination which reduces the time microorganisms spend in the bladder and by diluting the number of microorganisms in the bladder. Selfcatheterization helps prevent dribbling, so (C) is unnecessary. Cholinergic drugs improve bladder muscle tone and help with bladder emptying, not (D).

A client's daughter is sitting by her mother's bedside who was recently transferred to the Intermediate Care Unit. She states "I don't understand what a brain attack is. The healthcare provider told me my mother is in serious condition and they are going to run several tests. I just don't know what is going on. What happened to my mother?" What is the best response by the nurse? A) "I am sorry, but according to the Health Insurance Portability and Accounting Act (HIPAA), I cannot give you any information." B) "Your mother has had a stroke, and the blood supply to the brain has been blocked." C) "How do you feel about what the healthcare provider said?" D) "I will call the healthcare provider so he/she can talk to you about your mother's serious condition."

B) "Your mother has had a stroke, and the blood supply to the brain has been blocked." Rationale: The nurse can discuss what a diagnosis means. Nancy is unable to make decisions, so the next of kin, her daughter, Gail, needs sufficient information to make informed decisions. The nurse has the knowledge, and the responsibility, to explain Nancy's condition to Gail. The nurse should give facts first, and then address her feelings after the information is provided

In assessing cancer risk, the nurse identifies which woman as being at greatest risk of developing breast cancer? A) A 35-year-old multipara who never breastfed. B) A 50-year-old whose mother had unilateral breast cancer. C) A 55-year-old whose mother-in-law had bilateral breast cancer. D) A 20-year-old whose menarche occurred at age 9.

B) A 50-year-old whose mother had unilateral breast cancer. The most predictive risk factors for development of breast cancer are over 40 years of age and a positive family history (occurrence in the immediate family, i.e., mother or sister). Other risk factors include nulliparity, no history of breastfeeding, early menarche and late menopause. Although all of the women described have one of the risk factors for developing breast cancer, (B) has the greater risk over (A,C and D).

The nurse is taking a history of a newly diagnosed Type 2 diabetic who is beginning treatment. Which subjective information is most important for the nurse to note? A) A history of obesity. B) An allergy to sulfa drugs. C) Cessation of smoking three years ago. D) Numbness in the soles of the feet.

B) An allergy to sulfa drugs. An allergy to sulfa drugs may make the client unable to use some of the most common antihyperglycemic agents (sulfonylureas). The nurse needs to highlight this allergy for the healthcare provider. (A) is common and warrants counseling, but does not have the importance of (B). (C) does increase the risk for vascular disease, but it is not as important to the treatment regimen as (B). Diabetic neuropathy, as indicated by (D), is common with diabetics, but when the serum glucose is decreased, new onset numbness can possibly improve.

The nurse is assessing a client's laboratory values following administration of chemotherapy. Which lab value leads the nurse to suspect that the client is experiencing tumor lysis syndrome (TLS)? A) Serum PTT of 10 seconds. B) Serum calcium of 5 mg/dl. C) Oxygen saturation of 90%. D) Hemoglobin of 10 g/dl.

B) Serum calcium of 5 mg/dl. TLS results in hyperkalemia, hypocalcemia, hyperuricemia, and hyperphosphatemia. A serum calcium level of 5 (B), which is low, is an indicator of possible tumor lysis syndrome. (A, C, and D) are not particularly related to TLS.

The nurse is caring for a client with an oxygen saturation of 88% and accessory muscle use. The nurse provides oxygen and anticipates which of these physician orders? A) Administration of IV sodium bicarbonate B) Computed tomography (CT) of the chest, stat C) Intubation and mechanical ventilation D) Administration of concentrated potassium chloride solution

C) Intubation and mechanical ventilation Rationale: Support with mechanical ventilation may be needed for clients who cannot keep their oxygen saturation at 90% or who have respiratory muscle fatigue. A) Sodium bicarbonate is used to treat metabolic acidosis; this client displays hypoxemia. B) Although the underlying reason for this client's hypoxemia may eventually require a diagnostic study, the priority is to restore oxygenation. D) No indication suggests that this client has hypokalemia. Signs of hypoxemia and work of breathing are present, requiring correction with intubation and mechanical ventilation.

A nurse is preparing to insert an IV catheter after applying an eutectic mixture of lidocaine and prilocaine (EMLA), a topical anesthetic cream. What action should the nurse take to maximize its therapeutic effect? A) Rub a liberal amount of cream into the skin thoroughly. B) Cover the skin with a gauze dressing after applying the cream. C) Leave the cream on the skin for 1 to 2 hours before the procedure. D) Use the smallest amount of cream necessary to numb the skin surface.

C) Leave the cream on the skin for 1 to 2 hours before the procedure. Topical anesthetic creams, such as EMLA, should be applied to the puncture site at least 60 minutes to 2 hours before the insertion of an IV catheter (C). (A, B, and D) do not ensure a therapeutic response.

A college student who is diagnosed with a vaginal infection and vulva irritation describes the vaginal discharge as having a cottage-cheese appearance. Which prescription should the nurse implement first? A) Cleanse perineum with warm soapy water 3 times per day. B) Instill the first dose of nystatin (Mycostatin) vaginally per applicator. C) Perform glucose measurement using a capillary blood sample. D) Obtain a blood specimen for sexually transmitted diseases (STDs).

Correct Answer(s): B Candidiasis, also known as a yeast infection, is characterized by a white, vaginal discharge with a cottage-cheese appearance and vaginal nystatin (Mycostatin) should be implemented first (B) to initiate treatment to provide relief of symptoms. (A, C, and D) may implemented after (B).

The nurse is caring for a client after a transurethral resection of the prostate and determines the client's urinary catheter is not draining. What should the nurse implement? A) Reposition the catheter drainage tubing. B) Encourage the client to drink oral fluids. C) Irrigate the catheter. D) Change drainage unit tubing.

Correct Answer(s): C Obstruction urinary flow after a TURP is most often due to blood clots, and sterile irrigation should be implemented to remove the clots that are blocking the catheter (C). (A and B) will not relieve the obstruction at this time. (D) is not necessary.

The nurse is providing instructions about log rolling to a client who returns to the postoperative unit after a lumbar laminectomy. Which explanation should the nurse give the client about this technique? A) Helps to minimize pain and anxiety. B) Maintains correct spinal alignment to protect the surgical area. C) Prevents dizziness while stabilizing the spine. D) Allows the nurse to move the client freely without assistance.

Correct Answer: B Log-rolling technique maintains the spine in a straight superior-inferior plane that aligns the spine without movement while protecting the surgical area (B), which is especially important when the procedure involves bone grafts that may several weeks for the bone to fuse. (A) is best managed with client teaching, preparation before procedures, relaxation techniques, and pain management, not log-rolling. (C and D) are not accurate.

An older adult client begins wearing binaural hearing aids due to presbycusis. Which instruction should the nurse provide to assist the client in adapting to the new hearing aids? A) Begin wearing the aids in quiet environments to experiment with adjustments. B) Wear the hearing aids for an hour a day at first, gradually increasing the time. C) Keep the volume on low until the conditions with noises are audible. D) Use one hearing aid until comfortable, then add the second aid.

Correct Answer:A

A client with diabetes mellitus is experiencing polyphagia. Which outcome statement is the priority for this client? A) Fluid and electrolyte balance. B) Prevention of water toxicity. C) Reduced glucose in the urine. D) Adequate cellular nourishment.

D) Adequate cellular nourishment. Diabetes mellitus Type 1 is characterized by hyperglycemia that precipitates glucosuria and polyuria (frequent urination), polydipsia (excessive thirst), and polyphagia (excessive hunger). Polyphagia is a consequence of cellular malnourishment when insulin deficiency prevents utilization of glucose for energy, so the outcome statement should include stabilization of adequate cellular nutrition (D). (A, B, and C) relate to subsequent osmolar fluid shifts related to glucosuria, polyuria, and polydipsia.

A client experiencing uncontrolled atrial fibrillation is admitted to the telemetry unit. What initial medication should the nurse anticipate administering to the client? A) Xylocaine (Lidocaine). B) Procainamide (Pronestyl). C) Phenytoin (Dilantin). D) Digoxin (Lanoxin).

D) Digoxin (Lanoxin). Digoxin (Lanoxin) (D) is administered for uncontrolled, symptomatic atrial fibrillation resulting in a decreased cardiac output. Digoxin slows the rate of conduction by prolonging the refractory period of the AV node, thus slowing the ventricular response, decreasing the heart rate, and effecting cardiac output. (A, B, and C) are not indicated in the initial treatment of uncontrolled atrial fibrillation.

A client is admitted to the emergency department after being lost for four days while hiking in a national forest. Upon review of the laboratory results, the nurse determines the client's serum level for thyroidstimulating hormone (TSH) is elevated. Which additional assessment should the nurse make? A) Body mass index. B) Skin elasticity and turgor. C) Thought processes and speech. D) Exposure to cold environmental temperatures.

D) Exposure to cold environmental temperatures. TSH influences the amount of thyroxine secretion which increases the rate of metabolism to maintain body temperature near normal. Prolonged exposure to cold environmental temperatures (D) stimulates the hypothalamus to secrete thyrotropin-releasing hormone, which increases anterior pituitary serum release of TSH. (A) may reflect weight loss from lack of food. Tenting of the skin (B) is indicative of dehydration. Slow or confused thought processes (C) or speech patterns may be related to sleep deprivation

Despite several eye surgeries, a 78-year-old client who lives alone has persistent vision problems. The visiting nurse is discussing painting the house with the client. The nurse suggests that the edge of the steps should be painted which color? A) Black. B) White. C) Light green. D) Medium yellow.

D) Medium yellow. Yellow is the easiest for a person with failing vision to see (D). (A) will be almost impossible to see at night because the shadows of the steps will be too difficult to determine, and would pose a safety hazard. (B) is very hard to see with a glare from the sun and it could hurt the eyes in the daytime to look at them. (C) is a pastel color and is difficult for elderly clients to see.

How should the nurse position the electrodes for modified chest lead one (MCL I) telemetry monitoring? A) Positive polarity right shoulder, negative polarity left shoulder, ground left chest nipple line. B) Positive polarity left shoulder, negative polarity right chest nipple line, ground left chest nipple line. C) Positive polarity right chest nipple line, negative polarity left chest nipple line, ground left shoulder. D) Negative polarity left shoulder, positive polarity right chest nipple line, ground left chest nipple line..

D) Negative polarity left shoulder, positive polarity right chest nipple line, ground left chest nipple line. In MCL I monitoring, the positive electrode is placed on the client's mid-chest to the right of the sternum, and the negative electrode is placed on the upper left part of the chest (D). The ground may be placed anywhere, but is usually placed on the lower left portion of the chest. (A, B, and C) describe incorrect placement of electrodes for telemetry monitoring

The nurse obtains a client's history that includes right mastectomy and radiation therapy for cancer of the breast 10 years ago. Which current health problem should the nurse consider is a consequence of the radiation therapy? A) Asthma. B) Myocardial infarction. C) Chronic esophagitis with gastroesophageal reflux. D) Patholoic fracture of two ribs on the right chest.

D) Pathologic fracture of two ribs on the right chest. The ribs lie in the radiation pathway and lose density over time, becoming thin and brittle, so the occurence of two right-sided ribs with pathological fractures resulting without evidence of trauma (D) is related to radiation damage. The heart (B), esophagus (C), and larger bronchi (A) are not usually in the radiation path.

What instruction should the nurse give a client who is diagnosed with fibrocystic changes of the breast? A) Observe cyst size fluctuations as a sign of malignancy. B) Use estrogen supplements to reduce breast discomfort. C) Notify the healthcare provider if whitish nipple discharge occurs. D) Perform a breast self-exam (BSE) procedure monthly.

D) Perform a breast self-exam (BSE) procedure monthly. Fibrocystic changes in the breast are related to excess fibrous tissue, proliferation of mammary ducts and cyst formation that cause edema and nerve irritation. These changes obscure typical diagnostic tests, such as mammography, due to an increased breast density. Women with fibrocystic breasts should be instructed to carefully perform monthly BSE (D) and consider changes in any previous "lumpiness." Fibrocystic disease does not increase the risk of breast cancer (A). Cyst size fluctuates with the menstrual cycle, and typically lessens after menopause, and responds with a heightened sensitivity to circulating estrogen (B), which is not indicated. Nipple discharge associated with fibrocystic breasts is often milky or watery-milky and is an expected finding (C).

The healthcare provider prescribes aluminum and magnesium hydroxide (Maalox), 1 tablet PO PRN, for a client with chronic renal failure who is complaining of indigestion. What intervention should the nurse implement? A) Administer 30 minutes before eating. B) Evaluate the effectiveness 1 hour after administration. C) Instruct the client to swallow the tablet whole. D) Question the healthcare provider's prescription.

D) Question the healthcare provider's prescription. Magnesium agents are not usually used for clients with renal failure due to the risk of hypermagnesemia, so this prescription should be questioned by the nurse (D). (A, B, and C) are not recommended nursing actions for the administration of aluminum and magnesium hydroxide (Maalox).

A client with gastroesophageal reflux disease (GERD) has been experiencing severe reflux during sleep. Which recommendation by the nurse is most effective to assist the client? A) Losing weight. B) Decreasing caffeine intake. C) Avoiding large meals. D) Raising the head of the bed on blocks.

D) Raising the head of the bed on blocks. Rationale: Raising the head of the bed on blocks (D) (reverse Trendelenburg position) to reduce reflux and subsequent aspiration is the most effective recommendation for a client experiencing severe gastroesophageal reflux during sleep. (A, B and C) may be effective recommendations but raising the head of the bed is more effective for relief during sleep

A client with cirrhosis develops increasing pedal edema and ascites. What dietary modification is most important for the nurse to teach this client? A) Avoid high carbohydrate foods. B) Decrease intake of fat-soluble vitamins. C) Decrease caloric intake. D) Restrict salt and fluid intake.

D) Restrict salt and fluid intake. Salt and fluid restrictions are the first dietary modifications for a client who is retaining fluid as manifested by edema and ascites (D). (A, B, and C) will not impact fluid retention.

The nurse is completing the health assessment of a 79- year-old male client who denies any significant health problems. Which finding requires the most immediate follow-up assessment? A) Kyphosis with a reduction in height. B) Dilated superficial veins on both legs. C) External hemorrhoids with itching. D) Yellowish discoloration of the sclerae.

D) Yellowish discoloration of the sclerae. Jaundice, a yellowish discoloration of the sclerae (D), may indicate liver damage and requires further assessment. Kyphosis and height reduction (A) due to bone loss, varicose veins (B), and external hemorrhoids with itching (C) are common findings in the elderly that do not require immediate intervention.

Dysrhythmias are a concern for any client. However, the presence of a dysrhythmia is more serious in an elderly person because A) elderly persons usually live alone and cannot summon help when symptoms appear. B) elderly persons are more likely to eat high-fat diets which make them susceptible to heart disease. C) cardiac symptoms, such as confusion, are more difficult to recognize in the elderly. D) elderly persons are intolerant of decreased cardiac output which may result in dizziness and falls.

D) elderly persons are intolerant of decreased cardiac output which may result in dizziness and falls. Cardiac output is decreased with aging (D). Because of loss of contractility and elasticity, blood flow is decreased and tachycardia is poorly tolerated. Therefore, if an elderly person experiences dysrhythmia (tachycardia or bradycardia), further compromising their cardiac output, they are more likely to experience syncope, falls, transient ischemic attacks, and possibly dementia. Most elderly persons do not eat high-fat diets (B) and most are not confused (C). Although many elderly persons do live alone, inability to summon help (A) cannot be assumed.

A 77-year-old female client is admitted to the hospital. She is confused, has no appetite, is nauseated and vomiting, and is complaining of a headache. Her pulse rate is 43 beats per minute. Which question is a priority for the nurse to ask this client or her family on admission? "Does the client A) have her own teeth or dentures?" B) take aspirin and if so, how much?" C) take nitroglycerin?" D) take digitalis?"

D) take digitalis?" Elderly persons are particularly susceptible to digitalis intoxication (D) which manifests itself in such symptoms as anorexia, nausea, vomiting, diarrhea, headache, and fatigue. Although it is important to obtain a complete medication history (B and C), the symptoms described are classic for digitalis toxicity, and assessment of this problem should be made promptly. (A) is irrelevant.

A client was admitted with the diagnosis of a brain attack. Their symptoms began 24 hours before being admitted. Why would this client not be a candidate for thrombolytic therapy?

Thrombolytic therapy is contraindicated in clients with symptom onset longer than 3 hours prior to admission. This client had symptoms for 24 hours before being brought to the medical center


संबंधित स्टडी सेट्स

Individual Health Insurance Policy Provisions 14

View Set

Interpersonal Comm Final (Ch. 1-4; 9-12)

View Set

AP Psychology Chapter 2: Research Methods Multiple Choice Part 2/2

View Set

Intro to Combustible Dust Hazards

View Set

Strategic Management Exam 1 Practice

View Set

Converting yards, feet, and inches

View Set